Incorrect Psychology Questions

Réussis tes devoirs et examens dès maintenant avec Quizwiz!

Which of the following plays the most prominent role in the Cannon-Bard Theory of emotion? A) Thalamus B) Hypothalamus C) Amygdala D) Dopamine

B. The role of the hypothalamus is central in Cannon-Bard theory, which is concerned with that region's regulation of emotion and physiological response (choice B is correct). The thalamus, amygdala, and dopamine are of peripheral interest in the Cannon-Bard theory (choice A, choice C, and choice D are wrong).

Suzanne was in the middle of deep sleep and was suddenly jolted awake and alert by her alarm clock. Most likely, Suzanne's brain wave state has quickly shifted from: A) alpha wave to delta wave. B) delta wave to beta wave. C) theta wave to alpha wave. D) beta wave to theta wave.

B. This question calls for knowledge of four brain wave states. Beta waves are associated with alert wakefulness, while alpha waves are associated with relaxed wakefulness or drowsiness. Theta waves are active during drowsiness or sleep, and delta waves are associated with deep sleep. The question described Suzanne as being jolted from deep sleep to alert wakefulness, which corresponds to a shift from delta wave to beta wave (choice B is correct; choices A, C, and D are wrong).

It is possible that gentrification contributes to black-white residential segregation at the expense of minorities through which of the following mechanisms? This process restructures local neighborhoods such that the new structure better supports those with higher incomes. This process causes rising housing prices such that long-term residents cannot manage the costs. This process increases the tax base needed to provide essential services in white neighborhoods but not black neighborhoods. A) I and II only B) I and III only C) II and III only D) I, II, and III

A. Item I is true: There is an important socioeconomic component to both gentrification and residential segregation. The process of gentrification tends to benefit the higher income newcomers who are responsible for the urban renewal, which can then contribute to the displacement of the lower income native residents in the urban neighborhood. There is a significant relationship between socioeconomic status (SES) and race in the United States and it is indeed true that minorities in the United States tend to be SES-disadvantaged compared to whites (choice C can be eliminated). This is a fundamental to the concept of gentrification; residential shifts most often have negative effects for native residents of color and positive effects for new higher income whites. Item II is true: The rising housing prices are one of the main results of the process described in the explanation of the first item (choice B can be eliminated). Item III is false: There are economic advantages associated with gentrification as discussed in the passage. This, however, does not explain the mechanism through which gentrification contributes to residential segregation (choice D can be eliminated and choice A is the correct answer). Furthermore, the increase in higher income residents in urban populations does contribute to the tax base of the effected cities, but this doesn't necessitate that the consequences are limited to the higher income neighborhoods. The higher tax base in these cities should be distributed to the services needed in each neighborhood where taxes are paid.

Two biological brothers, James and Paul, are eight and six years old, respectively. Lately, James has seemed much more advanced than his younger brother, which their mother attributes to birth order. She assumes that James works harder because he is the first born, while Paul seems to think the world revolves around him. However, when she takes the children to a psychologist, he suggests that their behaviors are instead reflective of Piaget's stages of cognitive development. Which of the following is true of the boys' stages of cognitive development? A) James is in the formal operations stage, and Paul is in the pre-operations stage. B) James is in the formal operations stage, and Paul is in the sensory motor stage. C) James is in the concrete operations stage, and Paul is in the pre-operations stage. D) James is in the concrete operations stage, and Paul is in the sensory motor stage

C. Piaget described four stages of cognitive development: the sensory motor stage, the pre-operations stage, the concrete operations stage, and the formal operations stage. The sensory motor stage generally occurs when children are between zero and two years old (choices B and D can be eliminated). The pre-operations stage generally occurs when children are between two and seven years old. At this stage, children tend to be egocentric, viewing the world only from one perspective. Based on the question stem, it can be concluded that Paul is in the pre-operations stage. Next, the concrete operations stage occurs when children are about seven to eleven years old. Finally, the formal operations stage usually occurs when children are aged twelve or older (choice A can be eliminated). Since James is eight, it can be determined that he is in the concrete operations stage (choice C is correct).

Children learn their roles in the world from their parents and other social influences. Roles and rules vary across families and cultures. For instance, spitting may be deemed offensive in one household but acceptable in another. Specific norms are therefore created by individuals and would not exist in the absence of human beings. This idea that people develop meanings of the world and thereby agree upon values is grounded in the theory of: A) symbolic interactionism. B) functionalism. C) conflict theory. D) social constructionism.

D. Social constructionism is concerned with the ways in which people understand, interpret, and give meaning to the world. It explains how human beings socially construct such aspects of life as roles and rules. Therefore, the same action, such as spitting, can be interpreted differently by different groups of people. The taboo, however, would not exist if humans had not created it in the first place (choice D is correct). Symbolic interactionism studies small scale interactions between and among human beings. It argues that people act based on meanings they have assigned to objects; but such meanings are fluid and subject to change. While symbolic interactionism is very similar to social constructionism, it is important to note that only the former focuses on the way people act (choice A is wrong). Functionalism involves multiple parts of an entire unit or society working together to establish equilibrium (choice B is wrong). Conflict theory refers to the ways in which societies transform over time. Oftentimes, two opposing viewpoints arise; constant competition leads to an ever-changing world. This perspective also focuses on social control and power struggles (choice C is wrong).

If the results showed that the participants in the Full Attention group scored higher than the Divided Attention group, this could have been caused by all of the following, EXCEPT: A) the second task was too long. B) the second task was too difficult. C) the second task was too similar. D) the second task required too much practice.

A. The length of a task is not one of the three main factors that determine whether a task will drain a person of resources when trying to multitask, since the resource model states that humans have a limited amount of resources available when trying to attend to multiple tasks. If the task was too long, it would not have necessary led to the FA group scoring higher (choice A is correct). Task difficulty is one of three factors associated with performance on multitasking according to the resource model. If the task was too difficult, the FA group might have scored higher (choices B is wrong). Again, if the second task was too similar to the first, the FA group might also have scored higher than the DA group (choice C is wrong). Similarly, if the second task required way too much practice, the FA group might have scored higher than the DA group (choice D is wrong).

The rules which govern appropriate emotional responses vary from culture to culture. These social mores are called: A) display rules. B) emotional expression. C) rituals. D) cultural emotions.

A. The rules governing emotional displays within a particular culture are called display rules (choice A is correct). Emotional expression is what is being regulated by the display rules (choice B is wrong). Rituals are a series of set activities, which can include gestures or words, that occur in a set place and in a set order; the social rules governing the expression of emotions do not (generally) address the order or place of the emotional display (choice C is wrong). Emotional responses are the experience of emotions, which is thought to be innate; there is no such thing as "cultural emotions," per se (choice D is wrong).

The way in which children appear to be acquiring and practicing language skills through the methods described in the research above suggest they are in what stage of language acquisition? A) Egocentric speech B) Social speech C) Inner speech D) Private speech

B. Social speech describes the first stage of speech in which children learn by imitating others and by learning to interact with others (choice B is correct). Egocentric speech is the next phase, where children begin to talk to themselves and experiment with language usage (choice A is wrong). Inner speech is the third stage, in which children internalize language development and have an inner sense of grammar and syntax, so that the child no longer needs to practice them aloud (choice C is wrong). Private speech is another term for egocentric speech, the second phase in which children are practicing their language skills aloud but to themselves (choice D is wrong).

Alzheimer's Disease is known to affect cognitive centers of the brain at all stages: which of the following is not usually a symptom of early-onset Alzheimer's? A) Sleep apnea B) Cognitive impairment C) Primary progressive aphasia D) Short-term memory loss

A. The question stem states that Alzheimer's disease affects cognitive centers. Sleep apnea is a somatic condition, not a cognitive one (choice A is correct). Cognitive impairment in any noticeable sense is one of the first detectable symptoms of Alzheimer's, and this would represent an impairment of cognitive centers (choice B is wrong). Primary progressive aphasia is a rare disease that results in impaired language ability, and language is an ability governed by cognitive centers (choice C is wrong). Short term memory loss is one of the most common symptoms of Alzheimer's Disease and memory is a cognitive function (choice D is wrong).

Wernicke's area would be associated primarily with which of the following activities? A) Decoding the text B) Circling the ratings on the IITQ C) Asking a research assistant for help D) Comprehending the text

D. Wernicke's area is a region in the left hemisphere of the brain responsible for language comprehension (choice D is correct). It is not responsible for decoding, which refers to pronouncing the sounds of letters and words (choice A is wrong), nor is it associated with motor activity (choice B is wrong). Broca's area, not Wernicke's area, is associated with language production (choice C is wrong).

In the semantic network model, what determines the strength of a connection between a node and an association? A) How frequently and deeply connections are made B) How closely the node and association are explicitly related C) How many links there are between the node and the association D) How many state-dependent cues have been created between the node and the association

A. According to the semantic network model, the strength of a connection between a node and an association is related to how frequently and how deeply the connections are made (choice A is correct). The explicit relationship between the node and the association is meaningless unless that connection is constantly reinforced (choice B is wrong). Separate links between the node and the association—suggesting a more circuitous route—is indicative of the spreading activation pattern (choice C is wrong). State-dependent cues, in which familiar locales are used to trigger memories, do not determine the strength of the connection between a node and an association unless again, they are used with great frequency (choice D is wrong).

What would Albert Bandura suggest as one of the reasons why the group that watched the videotaped training scored higher than the control group or the group that was trained in the dialogic technique trained face-to-face? A) The group that trained with the videotape could model their behavior from the trainer on the tape. B) The group that trained with the videotape could watch the tape at home and thus were more relaxed and receptive to its teachings. C) The group that trained with the videotape could pause the tape when they wanted to take notes, a luxury denied people trained face-to-face in real time. D) The group that trained with the videotape preferred the direct instruction technique.

A. Albert Bandura proposed social learning theory, which includes learning via modeling: when a person sees another person perform a behavior and then imitates that behavior. Having a single trainer on a videotape that could be played repeatedly gave participants the opportunity to model the trainer's behavior and learn the dialogic reading strategies via modeling (choice A is correct). While it is possible that participants who watched the tape at home were more relaxed and receptive to its teaching, this does not best explain what Albert Bandura might suggest as one of the reasons why the group that watched the videotaped training scored higher (choice B is wrong). Similarly, while it is possible that participants in the videotape condition may have been able to take notes, this also does not best explain what Albert Bandura might suggest as one of the reasons why the group that watched the videotaped training scored higher (choice C is wrong). According to the passage, the face-to-face training group received direct instruction, while the videotaped training group received demonstrative training, according to the third paragraph of the passage (choice D is wrong).

A mother wonders whether her son has developed a specific phobia. Her son seems to become very anxious whenever he is at school. However, in reality, there are many different issues at school that are causing him anxiety. He is often worried about what he is wearing, his grades on past exams, and whether his fellow classmates enjoy his presence, but he has never suffered from a panic attack. Which of the following disorders is most likely the source of the boy's anxiety? A) General anxiety disorder B) Social phobia C) Post-traumatic stress disorder D) Bipolar disorder

A. An individual with general anxiety disorder experiences anxiety during much of the day due to a variety of issues (choice A is correct). A person with social phobia is afraid of situations in which he or she may experience humiliation (choice B is wrong). Post-traumatic stress disorder arises when a person feels intense fear following a traumatic experience or event (choice C is wrong). Bipolar disorder is a mood disorder and therefore does not explain the boy's anxiety (choice D is wrong).

A researcher wants to challenge the hypothesis that OXTR polymorphism genotype moderates the effect of social support on stress reactivity. The researcher proposes that individuals with GG genotypes were therefore raised by parents with either GG or AG genotypes and that the effect is actually environmental. What is one way that this researcher might test her hypothesis? A) Twin study B) Adoption study C) Transgenesis D) Developmental study

adoption study is the best way to do this. In an adoption study, researchers compare individuals with their genetic and adopted relatives to determine if they are more similar to one group or the other. If the researcher found that the effect shown in Figure 1 only persisted if individuals with the GG/AG genotypes were also raised by adopted parents with GG/AG genotypes, then she would argue that the effect is due to environmental, rather than genetic, differences. However, if the researcher found the effect shown in Figure 1 for all adopted individuals, regardless of their adopted parents' genotypes, this would be evidence for an effect of genetics. Therefore, an adoption study would be the best way to test this hypothesis and choice B is correct. In a twin study, researchers assess whether monozygotic (i.e. identical) twins are more similar than dizygotic (i.e. fraternal) twins to determine to what extent a behavior can be attributed to shared genes and to what extent it can be attributed to shared environment (choice A is wrong). Transgenesis is when a researcher either introduces a new gene or removes a gene from an animal's pre-existing genome in order to assess the effect of that gene in a tightly controlled experiment. Due to ethical concerns, it is highly unlikely that a human would ever be a participant in a transgenesis study (choice C is wrong). A developmental study would allow the researcher to assess the effect of genotype on behavior at many different points throughout a person's lifespan, but it would not allow her to assess whether the effect was due to entirely genetic or partially environmental influence (choice D is wrong).

A individual's description of her behavior during her job earlier that day is an example of retrieval from: A) episodic memory. B) semantic memory. C) procedural memory. D) olfactory memory.

A. Episodic memory refers to memory of one's personal experiences (choice A is correct). Semantic memory is memory of concepts and information (choice B is wrong), while procedural memory is memory of how to execute motor tasks (choice C is wrong). Olfactory memory is not a term associated with models of long term memory. In the context of short term memory, it would refer to a sensory store from which one could recall what a given stimulus smelled like. It would not have relevance to recall of one's behavior (choice D is wrong).

Which of the following is NOT a result or characteristic of high neuroticism? A) Increased extraversion B) More reactive sympathetic nervous systems C) Increased prevalence of mood disorders D) Prolonged states of negative mood

A. Extraversion and neuroticism are two separate personality traits in the Big Five (which also includes openness to experience, conscientiousness, and agreeableness); the two personality traits are unrelated; in other words, being high in neuroticism is unrelated to extraversion (choice A is correct). Individuals with high neuroticism often have overly active sympathetic nervous systems; this explain why they often display traits of hyper-arousal, which is correlated to aggressiveness and the tendency to anger easily (choice B can be eliminated). Neuroticism is correlated with an increased prevalence of mood disorders and prolonged states of negative mood (choices C and D can be eliminated).

The average age of participants in this meta-analysis was about 62 years old. That would place the average participant in which psychosocial crisis stage according to Erik Erikson's eight-stage theory of development? A) Integrity vs. Despair B) Generativity vs. Stagnation C) Identity vs. Role Confusion D) Trust vs. Mistrust

A. Integrity vs. Despair asks whether one can come to terms with the approach of one's own death or does one feel despair over the disappointments of one's life, the "natural" feelings of old age (choice A is correct). Generativity vs. Stagnation describes the period between one's 30s to one's 50s when a person struggles between building a career and family and getting stuck in routine (choice B is wrong). Identity vs. Role Confusion is the period between the ages of 12-19 when a teenager is finding him- or herself in the world (choice C is wrong). Trust vs. Mistrust is the period of the first year of life when a baby is deciding whether to trust his or her caregivers and environment (choice D is wrong).

Which of the following can be reasonably concluded from Figures 1 and 2? I. Caregivers of those with Alzheimer's and other dementias spend about a third of their time helping patients to and from the toilet, while caregivers for other older adults spend only about a quarter of their time helping patients to and from the toilet II. More than half of African-Americans and Hispanics over age 85 have AD III. Morbidity of Alzheimer's increases with age A) III only B) I and II only C) II and III only D) I, II, and III

A. Item I is false: The title for Figure 1 indicates that this figure is demonstrating the proportion (or percent) of caregivers who provide assistance with each daily living activity; therefore, 33% of caregivers for AD and other dementia patients help patients to and from the toilet, while 25% of other caregivers help with this activity. This figure demonstrates what percent of caregivers help with each activity, not what percent of time is spent on each activity (choices B and D can be eliminated). Item II is false: Because Figure 2 includes types of dementias other than just Alzheimer's, the conclusion that a majority of African-Americans and Hispanics over 85 have AD cannot be drawn without further information (choice C can be eliminated). Item III is true: According to Figure 2, the percentages of those afflicted with Alzheimer's disease and other dementias increases as the age cohort increases, for all three racial/ethnic groups (choice A is correct).

Which of the following statements can be inferred about role strain for women? Women tend to be under more stress than men in professional schools due to impression management. A female student in medical school is an example of front-stage self. A mother interacting with other mothers at the park is an example of back-stage self. A) I and II only B) I and III only C) II and III only D) I, II, and III

A. Item I is true: Since role strain is conflict resulting from having multiple identities, or playing multiple roles, it is related to impression management: people use impression management to craft their front-stage selves, and the difficulty of this can cause stress, known as role strain (choice C can be eliminated). Item II is true: A front-stage self in the dramaturgical perspective is the self the individual "performs" in front of an audience: the female's role as a student in medical school is performed in front of other members of society such as the other medical students, teachers, and staff (choice B can be eliminated). Item III is false: A back-stage self in the dramaturgical perspective is the self the individual is not performing in front of any audience: the woman in this scenario is still playing the role of the mother in front of other mothers, which is considered front-stage behavior (choice D can be eliminated and choice A is, therefore, the correct answer).

Which of the following is/are an accurate description of a big five personality traits? Extraversion: People high in extraversion tend to be outgoing, sociable, upbeat, and friendly. Conscientiousness: People high in conscientiousness tend to be curious, flexible, and imaginative. Kindness: People high in kindness tend to be sympathetic, trusting, and cooperative. A) I only B) II only C) I and II only D) I and III only

A. Item I is true: The big five traits are extraversion, neuroticism, openness to experience, agreeableness, and conscientiousness. Extraversion is one of the big five traits, and people high in extraversion tend to characterized as outgoing, sociable, upbeat, and friendly (choice B can be eliminated). Item II is false: Conscientiousness is one of the big five traits. However, this trait is associated with being diligent, disciplined, and well-organized. It is not associated with curiosity, flexibility, and imaginativeness (choice C can be eliminated). Item III is false: Kindness is not one of the big five traits (choice C can be eliminated, and choice A is correct).

How might a theorist in the symbolic interactionist tradition explain deviance? A) Deviant behavior is a response to being labeled a deviant by others. B) Deviant behavior results from performing on the back stage when one should be performing on the front stage. C) Deviance is a failure to develop a looking-glass self. D) Deviance is a failure to resolve the identity versus role confusion dilemma in adolescence.

A. Labeling theory is a classic symbolic interactionist approach. It states that deviance is not inherent in the act itself, but is rather an attribution from others (such as police, family, or clergy; choice A is correct). Back stage and front stage dynamics are an aspect of the dramaturgical approach. If one acts on the back stage when she/he should be performing on the front stage, the audience might experience more than it otherwise would (more details about a person's motivation or inner thoughts, for instance), but this would not necessarily be a deviant act (choice B is wrong). A looking-glass self describes how people shape their self-concepts based on how they think others perceive them (choice C is wrong). The identity versus role confusion dilemma is one stage in Erikson's eight developmental stages. This is not part of the symbolic interactionist tradition (choice D is wrong).

According to Lawrence Kohlberg, what is the highest level of morality that the typical average adult tends to achieve? A) She or he will acknowledge a sense of duty to uphold the laws of the land and maintain basic social conventions. B) He or she will strive for social justice—the greatest good for the greatest number. C) He or she will focus on the approval (and/or disapproval) of others, acting in accordance with how he or she will be viewed by his or her community or peer group. D) She or he will act in such a way as to avoid punishment at all costs

A. Lawrence Kohlberg postulated six stages of moral development, but said that the average person generally did not pass beyond the fourth stage, in which he or she acknowledged a sense of duty to uphold the law and maintain social conventions (choice A is correct). Striving for social justice, where one acts for the greatest good for the greatest number, was the fifth stage in Kohlberg's schema and one that he proposed that few people reach (choices B is wrong). A focus on receiving approval from one's peers along with a need to conform is the third stage of development, usually seen in older children and adolescents (choice C is wrong). Acting to avoid punishment is the earliest stage of moral development, typical of very young children (choice D is wrong).

Which of the following is a negative symptom of schizophrenia? A) Avolition B) Disorganized speech C) Hallucinations D) Delusions

A. Negative symptoms of schizophrenia are defined as deficits in normal behavior, such as avolition, or lacking any motivation to initiate behavior. Someone experiencing avolition will remain in the same position for long periods of time unless moved by someone else (choice A is correct). Positive symptoms of schizophrenia are those that most individuals do not normally experience but are present in people with schizophrenia. Disorganized speech is speech which lacks a goal or coherence; this is a positive symptom (choice B is wrong). A hallucination is the perception of sensation (feeling, hearing or seeing) in the absence of an external stimulus, such as hearing a voice when no one is speaking or seeing something that no one else sees; this is a positive symptom (choice C is wrong). Delusions are false beliefs held with conviction in spite of contradictory evidence; this is a positive symptom (choice D is wrong).

Emotion plays a critical role in attitude change. A great deal of attitude research notes the importance of affective/emotional components. Important factors to consider in emotion's impact on attitude are: self-efficacy, attitude accessibility, issue involvement and message/source features. Of these factors, which has the greatest potential to change the individual's emotional self-perception - the way they feel about themselves? A) Self-efficacy B) Attitude accessibility C) Issue involvement D) Message source/features

A. Self-efficacy is a perception of human agency, the perception of one's ability to face a situation. It dictates one's ability to deal with emotion brought up by a situation, and to process it via perception (choice A is correct). Attitude accessibility refers to the activation of an attitude from memory. Studies have shown that accessible attitudes are change-resistant (choice B is wrong). Issue involvement is the relevance and salience of an issue or situation to an individual; it is correlated with attitude access and attitude strength, both of which are distinct from perception (choice C is wrong). Message source and features of the message being sent are on an object-subject correlative; they are not directly relevant to perception (choice D is wrong).

The Levels of Processing theory would be supported by a finding that: A) participants in the low coherence condition performed better on a task assessing long term memory of the material than did participants in the high coherence condition, regardless of intelligence theory orientation. B) participants in the high coherence condition performed better on a task assessing long term memory of the material than did participants in the low coherence condition, regardless of intelligence theory orientation. C) entity theorists in the high coherence condition performed better than incremental theorists in the same condition on a problem solving task. D) incremental theorists in the high coherence condition showed lower emotional arousal in response to the text than did entity theorists in the same condition.

A. The Levels of Processing theory posits that people remember information better when they process it more deeply. For example, people who are asked to answer questions about material they read will remember the material better than those who simply read it. Since participants in the low coherence condition must think more deeply about the text in order to understand it, it follows that they will remember the material better and thus perform better on a long-term memory task. Additionally, while the researchers expected differences in perceived comprehension between the two types of theorists, no expected difference in depth of processing or actual comprehension is indicated by the passage (choice A is correct; choice B is wrong). The passage did not suggest that entity and incremental theorists would differ regarding problem solving ability (choice C is wrong) or emotional arousal (choice D is wrong). Further, neither problem solving nor emotional arousal is relevant to the Levels of Processing theory.

Current research on overall job satisfaction (Figure 1) and overall job-related stress among physician specialists demonstrates substantial differences among the specialties. In both figures, the five specialties with the highest and lowest reports are represented, as well as the mean response. Which of the following can be most reasonably concluded from these data? A) There is some evidence of a negative association between job satisfaction and job-related stress. B) There is some evidence of a positive association between job satisfaction and job-related stress. C) There is conclusive evidence of an association between job satisfaction and job-related stress. D) There is no evidence of an association between job satisfaction and job-related stress.

A. The data presented in the figures suggest a possible relationship between job satisfaction and job-related stress (choice D is wrong). Positive or direct correlations are defined as those in which two variables move in the same direction (e.g., increase together or decrease together). There is some evidence that specialists reporting lower job-related stress (e.g. preventative medicine, general pediatrics, and dermatology) also report higher job satisfaction; there is not enough information to conclude the two factors are negatively correlated, but they may be negatively associated (choice A is correct; choice B is wrong). However, the data provided is certainly not conclusive (choice C is wrong). For example, the opposite does not appear to be true; the three specialties reporting the lowest rates job satisfaction are not the three specialties reporting the highest rates of job-related stress. It is difficult to provide conclusive statements about the relationship between these two factors with the limited data presented; the figures provided represent only the five specialties at either extreme of the two spectrums.

Gender studies have been of interest to scholars of various academic disciplines, including sociologists. Those sociologists who adhere to the functionalist perspective would note that gender stratification and gender roles: A) exist to maximize social coherence; for example, men have instrumental social roles and women have expressive social roles. B) result from past traditions that prevented the advancement of women, traditions which began with the rise of capitalism. C) reflect the disadvantages of women as a result of power inequalities present in the social and economic structure. D) lead to inequalities in communication; for example, it is more acceptable for men to interrupt women during conversation.

A. The functionalist perspective, also called functionalism, is interested in structural processes which maintain social order. Based on this theoretical perspective, social functioning is possible through the independent contributions of individual social structures. Thus, it is expected that sociological theorists interested in functionalism would research the contributions of gender stratification to overall social cohesion (choice A is correct). Functionalists might propose that men and women have separate social roles, which together have social benefits. For example, it might be expected that men are represented in the workplace and that women are represented in the household, both of which contribute to social coherence. However, the remaining arguments are more predicted from scholars of opposing theoretical perspectives. Discussion of traditions preventing the advancement of women, and capitalism in particular, as well as discussion of power inequalities, is expected from conflict theorists (choices B and C are wrong). This perspective is interested in structural processes which maintain social inequalities. Discussion of communication, a micro-sociological concern, is expected from interactionists (choice D is wrong). This perspective is interested in processes of interaction that create social meaning.

The researchers' theoretical basis is most similar to which of the following models? A) Spreading activation model B) Set-theoretical model C) Dual-coding hypothesis model D) Level of processing model

A. The passage describes verbal insight problems as those containing words that tend to activate the wrong set of internal representations. The spreading activation model of verbal knowledge posits that the mind maintains networks of words that are associated with each other to varying degrees. When one word is activated, those words with the strongest associations to the activated word are also activated. Thus, the use of verbal insight problems that employ words leading to incorrect activations is strongly related to the spreading activation model (choice A is correct). The set-theoretical model holds that the mind contains sets of information that are systematically compared in order to retrieve the concept in question. This model emphasizes the comparison of category features rather than activation of words, so it is less similar to the framework used by the researchers (choice B is wrong). The Dual-Coding Hypothesis is a memory model that describes how some information is encoded visually, while other information is encoded primarily through an auditory process. It does not explain the mechanisms of verbal knowledge formation (choice C is wrong). The Levels of Processing maintains that long term memories are formed through more intensive processing of material, such as by thinking critically about it. However, it does not explain how verbal knowledge is activated (choice D is wrong).

Famous studies in the 1940s and 1950s found that infants living in orphanages had a high mortality rate, even when controlling for factors such as pre-existing conditions and medical treatment. The cause, according to these early studies, was the lack of human contact. These results, if true, would: A) bolster the results of the current study. B) dispute the results of the current study. C) have no effect on the results of the current study. D) only partially relate to the current study because that study was about physical contact and the current one is about social support.

A. The results of the previous research support the current research since physical contact is one way of showing social support—they are both forms of human interaction (choice A is correct). The other responses would indicate that need for social support in the early studies would contradict the need in the present studies, which is untrue (choice B is wrong), or that the results of the first would be have no effect on the second, which is also untrue (choice C is wrong). As noted above, physical contact is a way of showing social support (choice D is wrong).

Security personnel trained a dog to growl whenever a person appeared on the other side of a car lot fence by using a live cat as the unconditioned stimulus. After several trials, the dog growled every time a person approached the fence, even without the cat. At that point, use of the live cat was discontinued. After several weeks, the dog ceased to growl at the sight of a person nearing the fence. The owner took the dog to his home as a house pet for several weeks and then brought the dog back to the lot. Which of the following behaviors should the owner now expect from the dog? A) The dog will growl at the next person who approaches the fence. B) The dog will no longer growl, even at the sight of a live cat. C) The dog will growl continuously until an electric shock is administered. D) The dog will run away from the next person who approaches the fence

A. This question addresses what classical conditioning theorists call spontaneous recovery. The principle of spontaneous recovery is relevant after the process of extinction has occurred. In extinction, the conditioned stimulus is presented consistently without the unconditioned stimulus, and, eventually, the conditioned stimulus loses its excitatory power. However, if, after a delay wherein the conditioned stimulus is not presented either, the conditioned stimulus is presented again, it will elicit the conditioned response. In the case described in the question stem, the live cat is the unconditioned stimulus, the sight of a person approaching the fence is the conditioned stimulus, and growling at the approaching person is the conditioned response. The passage indicates that extinction has occurred when it states that use of the live cat was discontinued and the dog ceased to produce the conditioned response of growling. A delay is implied by the owner's taking the dog to his home. After the dog is brought back to the lot, according to the principle of spontaneous recovery, the dog should produce the conditioned response (growl) when next presented with the conditioned stimulus (person approaching the fence; choice A is correct). There is no principle of classical conditioning that posits that, given the scenario in the question stem, the unconditioned stimulus (cat) will lose its excitatory value (choice B is wrong) or that the conditioned stimulus will produce a new response (choice D is wrong). The use of an electric shock to stop growling behavior would constitute a positive punishment, which is an operant (not classical) conditioning principle (choice C is wrong).

One of the modifications Milgram made to his original shock experiments was to test the effect of conformity on obedience. The general experiment tests the conditions under which a research subject, the "Teacher," will administer an electric shock to the "Learner" (a confederate member the research team). The "Teacher" receives instructions from a Scientist (member of the research team) to administer the shock. Which of the following experimental modifications is an example of the effect of conformity on obedience? I. Adding another confederate "Teacher" to the shock administration room II. Adding a window into the room so the "Teacher" could see the "Learner" III. Increasing the proximity of the "Teacher" to the "Learner" A) I only B) II only C) I and II only D) II and III only

A. To test the relationship between conformity and obedience, we would be primarily interested in how the subject ("Teacher") obeys the research team members, and then secondarily, whether the "Teacher" conforms to the research team's actions. This means that the interactions of the research team members are of interest here, not the interactions between the "Teacher" and the "Learner" who receives the shock. Item I is true: adding another confederate "Teacher" to the shock administration room would mean that the two Teachers would obey instructions given by the scientist to administer shocks, and also that the "Teacher" would conform to the actions of the new confederate Teacher when deciding to administer shock. Item II is false: Adding a window to into the room where the learner will receive the shock will not test the effect of conformity on obedience because it does not add a figure for the subject "Teacher" to conform to—this is merely a modification of the test of obedience; it tests whether or not the subject "Teacher" still obey the scientist when she or he can see the victim of the shocks (choice B can be eliminated, choice C can be eliminated). Item III is false: similarly, merely increasing the proximity of the "Teacher" to the "Learner" will only test obedience; there is no other person present for the subject "Teacher" to conform to (choice D can be eliminated, choice A is correct).

It appears that in their original predictions, participant wives who reported steeper drops in marital satisfaction might have used which defense mechanisms? A) Displacement B) Denial C) Repression D) Reaction formation

B. According to Freud, people use ego defense mechanisms in order to protect themselves from the effects of anxiety. The results of the current research offer the following about the women who experienced the steepest declines in marital satisfaction: (1) these women reported higher levels of external stress and (2) following this, these women had more positive predictions for their marriage. The fact that these women were more optimistic about their futures despite the presence of significant external stressors suggests the presence of a cognitive bias (as described in the final paragraph) that offers self-protection. Denial is a defense mechanism that is most related to this situation (choice B is correct). Denial is the refusal to accept external realities that are threatening to the self; for example, the refusal to consider the effect external stressors could have on marital satisfaction. Displacement is the shift of emotions, such as aggressive or sexual impulses, to safer outlets (choice A is wrong). Repression is the psychological attempt to subdue emotionally painful memories (choice C is wrong). Reaction formation is expression of the opposite of one's feelings in order to conceal unacceptable or dangerous emotions (choice D is wrong).

Agoraphobia, as described in the passage, is a fear of circumstances in which escape is difficult. Which of the following is the correct subtype of agoraphobia? A) Natural environment B) Situational C) Animal D) Blood-injection-injury

B. Agoraphobics are afraid of situations in which it is hard to escape. Therefore, their fear depends on their circumstances and can be classified as situational (choice B is correct). Fears of natural phenomena, such as lightening, hurricanes, or tornadoes, would be classified as natural environment phobia (choice A is wrong). Phobias resulting from certain animals, such as arachnophobia, the fear of spiders, would have the animal subtype (choice C is wrong). Phobias that result from the fear of blood or medical procedures would be in the blood-injection-injury category (choice D is wrong).

How could the James-Lange theory of emotion best be explained by the dynamical systems theory concept of emergence? A) Perception of the emotional stimulus is determinative for the emotional experience B) Different collections of physiological markers combine to represent distinct emotions C) Convergence of perceptive stimuli onto the thalamic and hypothalamic structures creates a global emotional experience D) Cognition and physiology combine to create a more complex emotional experience than the constituent components

B. Emergence, according to the passage, describes the way a system composed of many separate elements combines these elements in such a way that the system itself is more complex than the constituent elements themselves. Since the James-Lange theory deals with the way different physiological states represent emotions, the James-Lange theory of emotion could be explained by the dynamic systems concept of emergence if different collections of physiological markers combine to represent distinct emotions (choice B is correct). Perception of the emotional stimulus being determinative does not bring together different constituent elements as in the definition of emergence, but is more similar to sensitivity to initial conditions (choice A is wrong). Convergence onto thalamic and hypothalamic structures is prominent in the Cannon-Bard theory, not the James-Lang theory (choice C is wrong). The way cognition and physiology combine during emotion is a central component of the Schachter-Singer, or two-factor, theory of emotion (choice D is wrong).

Attachment theory suggests that once an attachment style is established, it tends to persist into adulthood and old age. Which of the following interactions would be least likely to be affected by a person's established attachment style? A) The interaction between a father and his seven-year-old son on his first day at a new school. B) The interaction between a teenager and a cashier at the local supermarket. C) The interaction between a middle aged woman and her boss of fifteen years. D) The interaction between a newly wed couple.

B. Established attachment styles have been shown to persist into adulthood and old age. This framework affects bonds formed with romantic partners and significant others; in other words, people who play an important role in our daily lives. It can be inferred that the interaction between a teenager and a cashier at his/her local supermarket is not one that is of great significance in his/her daily life (choice B is the interaction that is least likely to be affected by a person's established attachment style, and is therefore correct). The interaction between a father and his seven year old on his first day of school would be influenced by both of their attachment styles. The child's attachment style would affect his comfort and ability to separate from his father and navigate and interact while at his new school. Likewise, the father's attachment style would affect his ability to comfortably let his child face this new time in his life without anxiety (choice A is wrong). The interaction between a middle aged woman and her boss of fifteen years would also be affected by the attachment style of both. Both are significant people in each other's lives due to the long-term relationship and bond formed throughout the fifteen years (choice C is wrong). The interaction between a newly wed couple would also be affected by the attachment style of both individuals, as their relationship will now be one of the most important and significant in both of their lives (choice D is wrong).

Parental resources are an important consideration in studies of child maltreatment. Research findings suggest that insufficient financial resources can lead to compromised parenting. This might explain the higher than expected rates of neglect in which set of families? A) Native families B) Traditional immigrant families C) Non-traditional immigrant families D) Immigrant families (both sets)

B. First, this question requires an understanding of the participant sample as described in the final paragraph. Based on their representation in the sample, native families should account for around 70 percent of the reports of neglect, traditional immigrant families for around 18 percent, and non-traditional immigrant families for around 12 percent. Next, using Figure 1, it is possible to determine whether each set of families is overrepresented or underrepresentation based on the sample distribution. Native families account for 85 percent of physical neglect cases (overrepresentation) and 67 percent of emotional and education neglect cases (underrepresentation). Traditional immigrant families account for 9 percent of physical neglect cases (underrepresentation) and 22 percent of emotional and educational neglect cases (overrepresentation). Non-traditional immigrant families account for 6 percent of physical neglect cases (underrepresentation) and 11 of emotional and educational neglect cases (underrepresentation). Based on the question stem, the correct answer should include those families that are overrepresented (choices C and D can be eliminated). Reports of neglect are higher than expected for both native families and traditional immigrant families in at least one area. Thus, the form of neglect must be considered. Native families are overrepresented in reports of physical neglect and traditional immigrant families are overrepresented in reports of emotional and educational neglect. Demographic research suggests that socioeconomic status is a determinant of social opportunities, such as education. Based on this trend, parental resources are more expected to lead to emotional or educational neglect (choice B is correct and choice A is wrong).

One aspect of socialization is learning the patterns of normal behavior of one's social group. Certain norms are formal and strongly enforced, while others are merely customary ways of doing things that don't carry harsh penalties when violated. These customary ways of doing things are known as: A) mores. B) folkways. C) values. D) sanctions

B. Folkways are conventional ways of acting within a certain culture. Violating them would make one look unusual, but the violation would not carry an official penalty (choice B is correct). Mores, on the other hand, are formal norms that are very strictly enforced because they protect basic tenets of society (treason, for example, violates mores of loyalty and patriotism; choice A is wrong). Values are a culture's standard for evaluating what is good and bad (choice C is wrong). Sanctions are the punishments for violating a norm (choice D is wrong).

Which of the following is NOT associated with a gestalt approach to visual perception problems? A) Symmetry B) Illusory contours C) Continuity D) Common fate

B. Gestalt psychology emphasizes the mental processes that perceive the "whole" of objects or situations. This school of psychology has been particularly influential in describing tendencies of human visual perception. Among these tendencies are symmetry, continuity, and common fate. Symmetry refers to the tendency to view objects with subtle asymmetries as symmetrical (choice A is true and therefore wrong), while continuity is the tendency to "see" a pattern continue even after it has physically terminated (choice C is true and therefore wrong). Finally, common fate is the tendency to visually group objects of the same category together (choice D is true and therefore wrong). The notion of illusory contours is a type of illusion whereby boundaries and forms are perceived by the mind even though they do not exist in the actual image. Though this is a tendency of human visual perception, it is not associated with the Gestalt school, per se (choice B is false and therefore correct).

How would Erik Erikson most likely respond to this study? A) Erikson would wholeheartedly agree with the premise of this study, as he believed that a person does not develop their full identity until they reach adulthood, usually between the ages of 18 to 35. B) Erikson would challenge the premise of this study, as he believed that identity formation was a question that adolescents dealt with primarily during the ages of 12 to 18. C) Erikson would agree with the premise of this study but disagree with its results, as he believed that one started formulating identity during the adolescent years but it is not until the later years—middle age and later in life—where one could really "know" oneself fully. D) Erikson would challenge the premise of this study, as he believed that identity formation was heavily influenced by unconscious factors.

B. In Erik Erikson's proposed stages of development, identity formation occurs during the adolescent years (ages 12 to 18). Consequently, Erikson would have most likely challenged the premise of this study, which implies that identity formation primarily occurs as adolescents transition to young adults (ages 18 to 22; choice B is correct). In Erikson's developmental stages, when humans are between the ages of 18 to 35, they are mostly concerned with creating intimate relationships, rather than creating their own individual identities (choice A is wrong). As noted above, Erikson would disagree with the premise of this study. He would also say that by the time one reached middle age or the later years, a person was less concerned with individual identity than they were with generativity vs. stagnation in middle age and integrity in the later years. Personal identity by that point is really a non-issue unless it plays a role in either generativity or integrity (choice C is wrong). While Erikson was in the psychoanalytic school of thought regarding personality development, Freud was more focused on the unconscious, while Erikson believed that social factors influenced development over a lifespan (choice D is wrong).

According to social cognitive theory, which of the following best represents a symbolic model from which a participant may have learned how to execute a shot on goal? A) A coach's demonstration B) A televised hockey game in which a player scored a goal C) The hand signals of the players in one of the video clips D) Step-by-step verbal instruction from a coach

B. In his social cognitive theory, Bandura details three types of models: live, symbolic, and verbal instruction. A symbolic model is a model that executes a task or exhibits a type of behavior or attitude from which a person can learn. Nevertheless, the symbolic model does not directly demonstrate that behavior or attitude to the learner, nor is it the intention of the symbolic model for the particular learner to learn that behavior or attitude. Examples of symbolic models include book characters, videogame characters, or televised athletes (choice B is correct). A live model is an individual who demonstrates a particular skill or behavior with the purpose of teaching the learner. Thus, a coach's demonstration is an example of a live model, not a symbolic model (choice A is wrong). By definition, step-by-step verbal instruction is an example of verbal instruction rather than of a symbolic model (choice D is wrong). Hand signals are a form of communication; they are not a symbolic model, nor are they an example of any of Bandura's model types (choice C is wrong).

Which of the below statements demonstrates the idea that mass media are agents of socialization? I. Young people's body image is affected by the bodies depicted in magazines II. News media coverage of sexual assault focuses primarily on the victim III. Social media enable online bullying A) I only B) I and II only C) II and III only D) I, II and III

B. Item I is true: The mass media present and perpetuate a body image that is valorized by society. Exposure to these valorized images can socialize young people to take on these body standards as ideal. This is a form of socialization (choice C can be eliminated). Item II is true: When news media coverage focuses primarily on the victim of the assault, women learn that it is their job to protect themselves from sexual assault, but perpetrators do not learn to stop assaulting women. Emphasizing women's victimhood in mass media is a form of gender socialization that carries with it messages about who to associate with, how much to drink, and how to dress oneself (choice A can be eliminated). Item III is false: While social media can be considered to be mass media, the actual socializing force in this case is that of the peer group; peer groups are merely using social media to socialize each other (choice B is correct, choice D can be eliminated).

Researchers used a probability-based sample to recruit non-physician participants, which lead to a non-physician sample representative of the national population. Therefore, even though demographic information is not presented in the passage, it is still possible to conclude that: the most common religious affiliation reported is Christian (when compared to Muslim, Jewish, Hindu, and Buddhist). the least common socioeconomic status reported is lower class (when compared to upper class, middle class, and working class). for married participants, the most common relationship structure reported is monogamous (when compared to polygamous). A) I and II only B) I and III only C) II and III only D) I, II, and III

B. Item I is true: The most common religious affiliation in the United States is Christianity (choice C can be eliminated). Christianity is the largest religion in the world, and in the United States, 80 percent of people identify as Christian. Item II is false: The least common socioeconomic status in the United States is upper class, not lower class (choices A and D can be eliminated). In the United States, the top 3 percent of the population is considered upper class while roughly a quarter of the population is considered in the lower class. Item III is true: The most common form of marriage in the United States is monogamy. In the United States, the social norm is monogamous marriages, in which the partners are married only to each other. Furthermore, this exception is reinforced through federal legislation: polygamous marriages are illegal in the United States (choice B is correct).

Suppose Subject 67 (quoted in the passage) is at a party for gay men and overhears a group of other men, including the party host, having a conversation about him with. The party host mentions that Subject 67 is bisexual and indicates that this would be a disadvantage to dating him; all the men in the group readily agree. Although one of the group members doesn't agree with the host and the others, he keeps his opinion to himself; this self-censorship to avoid "rocking the boat" is known as: A) mindguarding. B) social comparison. C) informational influence. D) group polarization.

B. Mindguarding is an aspect of groupthink, in which people self-censor in order to retain group harmony or avoid "rocking the boat" (choice A is correct). Social comparison is the process of evaluating one's opinion with regard to another's (choice B is wrong). Informational influence is similar to groupthink in that ideas that come into group discussion are those that tend to favor the dominant viewpoint. This, however, does not describe the deliberate self-censorship that this group member is engaging in, in order to stay in favor with the group (choice C is wrong). Group polarization is a process by which the existing views of the group are strengthened during group discussion (choice D is wrong).

Research has also considered the salaries of medical professionals, and the effects of compensation on overall life satisfaction. Reports of average earnings ranged from $156,000 to $315,000 with pediatricians reporting the lowest salaries and orthopedic surgeons and radiologists reporting the highest salaries. Based on this information, models of social stratification in the United States would consider the average anesthesiologist to be a member of which social class? A) Upper Class B) Middle Class C) Working Class D) Lower Class

B. Most sociologists consider socioeconomic status (SES) when discussing social stratification. This measure is often defined in terms of power, prestige, and wealth due to the common interrelationship among these factors. The reported salaries of specialists range from $157,000 to $316,000 for the listed lowest and highest earners, respectively. The average anesthesiologist is then expected to earn an intermediate amount (the research reports an annual average of approximately $310,000). While estimates differ, this earning potential alone suggests that anesthesiologists are middle class, perhaps upper middle class (choice B is correct). Middle class annual incomes range from $46,000 to the minimum earnings representing the upper class (this definition varies); 40 percent of the population is considered middle class. Despite the high earnings of these medical specialists in comparison to the national average, it is a common misconception that these professionals are "upper class"; according to most models of social stratification, the income range for upper class is much higher. Upper class annual incomes range from millions to billions; a mere 3 percent of the population are considered upper class (choice A is wrong). There are medical professionals who earn comparable amounts when accounting for additional income and wealth; for example, Dr. Mehmet Oz, a famous cardiothoracic surgeon who hosts a television program focusing on medical issues and personal health (estimated annual earnings of $4 million). However, the information provided considers the average anesthesiologist. Working class annual incomes range from $19,000 to $45,000; 30 percent of the population are considered working class (choice C is wrong). Lower class annual incomes range from no income to $18,000; 27 percent of the population are considered lower class (choice D is wrong).

According to Noam Chomsky, children learn all their vocabulary: A) solely from universal grammar. B) from a complex interaction of genetic and environmental factors. C) solely from environmental factors such as parents and teachers reading to them from an early age. D) from a genetic predisposition to good reading and vocabulary skills.

B. Noam Chomsky strongly believed that a complex interaction of genetic and environmental factors were involved in building children's vocabularies (choice B is correct), although the ability to learn language itself was innate (an idea that was eventually referred to as "universal grammar," a feature unique to humans that allowed them to develop language skills, but is not predictive of vocabulary; choice A is wrong). While Chomsky believed that environmental factors such as good schooling—good teachers and good educational programs—were important in developing children's vocabularies, he also believed that genetic factors played a role in vocabulary development and growth, since not all children in a class had equal vocabulary development (choice C is wrong). Conversely, Chomsky believed that genetics alone was insufficient in accounting for a child's vocabulary strength (choice D is wrong).

Which of the following is employed by many organisms in order to reduce the two-fold cost of sexual reproduction?

B. Pheromones are chemicals released by organisms that trigger a social response through detection by other members of the same species. They are often used as signals in sexual reproduction and to assess reproductive compatibility and fitness, reducing the two-fold cost (choice B is correct). Foraging is the search for wild food resources, which does not address the two-fold cost (choice A is wrong). Aggressive behavior towards other members of the species would not make it easier to find other individuals to mate with or help pass on a higher percentage of genes (choice C is wrong). Inclusive fitness may help increase an individual's likelihood of survival within a group, but this does not reduce either of the two parts of the Smith's two-fold cost: passing on less genes and challenges finding a mating partner (choice D is wrong).

The Whorfian hypothesis suggests that our perception of the world is based on our availability of language to describe it. If a language's only color words were white, black, red, and green, which of the following would a supporter of the Whorfian hypothesis believe? A) There is nothing purple in the environment of the native speakers of this language. B) Native speakers of this language will perceive grass and the ocean as being the same color. C) It is evolutionarily important for native speakers of this language to identify the color red, e.g., to avoid poisonous berries. D) Native speakers of this language are most likely colorblind.

B. Since the Whorfian hypothesis states that language drives cognition, a supporter would believe that the only distinctions between colors that a native speaker of this language would perceive are between black, white, red, and green. Therefore, they would not see a difference between the green of the grass and the blue of the ocean (choice B is correct). They would not necessarily believe that other colors are not present in the environment, just that they would accept that wavelength as one of the colors for which they have a term (choice A is wrong). The Whorfian hypothesis does not speak to why certain color terms are used above others, though there is a standard pattern described by Berlin and Kay (1969). It is irrelevant to the supporter whether the word red is an evolutionarily useful tool (choice C is wrong). If native speakers were colorblind and only used terms for the colors that they could see, this would contradict the Whorfian hypothesis, as cognition would be determining language, as opposed to the other way around (choice D is wrong).

A young man develops ophidiophobia, which is a fear of snakes. His fear is maintained because he intentionally avoids any encounters with snakes. Which of the following operant conditioning principles describes how the man is preserving his fear? A) Positive reinforcement B) Negative reinforcement C) Positive punishment D) Negative punishment

B. Since the avoidance of snakes is relieving the man's anxiety, he will continue to avoid snakes. Furthermore, a negative stimulus, the anxiety the man feels from a snake's presence, is "removed" (or avoided) following his avoidance behavior. Therefore, this set of behaviors describes negative reinforcement (choice B is correct). The man is not adding a positive stimulus following the avoidance behavior (choice A is wrong). Punishment describes a decrease in a behavior. The man's decrease in anxiety is not going to decrease the probability of the avoidance behavior (choices C and D are wrong).

There is a suggested relationship between social capital and international giving. This might be explained by the fact that: A) wealth can increase one's ability to provide financial support internationally. B) networks can increase one's awareness of international causes. C) education can increase one's understanding of abstract causes, such as human rights. D) spirituality can increase one's connections to others, both domestically and internationally.

B. Social capital refers to the idea that social networks have benefits. It is possible that these networks could, in turn, raise one's awareness of international causes, one of the three significant predictors of donations (choice B is correct). Wealth is an example of economic capital; education is an example of cultural capital; and spirituality is an example of spiritual capital (choices A, C, and D are wrong). Furthermore, while religiosity is referred to in the passage, and can, in fact, influence social networks and thus social capital, spirituality does not necessitate a relationship with a religious institution.

Are the results from the study presented in the passage supportive of the Information Processing Theory and why? A) Yes, because the individuals with GAD are more likely to experience anxiety through rumination, and the individuals with depression are more likely to experience anxiety through worry. B) Yes, because the individuals are engaging in cognitive processing through the use of cognitive strategies that is in response to environmental stimuli. C) No, because the researchers were not able to evaluate the types of cognitions produced by the inductions. D) No, because the individuals are not engaging in cognitive processing that is in response to environmental stimuli.

B. The Information Processing Theory states that humans process the information they receive with their cognitions, rather than merely responding behaviorally to stimuli or internal drives. In the results of the study, the researchers are purposefully inducing different types of cognitions that are eliciting a response from the individuals. It can be clearly seen from Figure 1, that in response to the worry induction and the rumination induction, individuals are reacting differently than what they are in the control condition. It can therefore be concluded that yes, individuals are using their cognitions and cognitive strategies (though maladaptive) to interact with the environmental stimuli presented in the research conditions (choice B is correct). Choice A is not appropriate because the graph states that individuals with GAD are more likely to experience anxiety rumination, not the opposite (choice A is wrong). The researchers did evaluate the types of cognitions experienced by the participants during the inductions (choice C is wrong). Likewise, the participants were engaging in cognitive processing in response to environmental stimuli (choice D is wrong).

A researcher attempts to explain the phenomenon described in the passage that couples who reported greater relationship satisfaction had greater physiological co-regulation and concludes that the cognitive interpretation of the physiological state of both partners led to their reported feelings of satisfaction. This is most similar to which of the following? A) Emotional intelligence B) Schachter-Singer theory C) Cannon-Bard theory D) James-Lange theory

B. The Schachter-Singer theory posits that emotional experience is determined by physiological state and the cognitive interpretation of that state. This is most similar to the description in the question stem, in which couples experience physiological co-regulation and cognitive interpretation (choice B is correct). Emotional Intelligence is the ability to control, interpret, and understand one's own emotions and the emotions of others. This is not as specific as choice B in capturing the information in the question stem (choice A is wrong). The Cannon-Bard theory focuses on the central role of the hypothalamus in regulating emotions and is less specifically concerned with physiological activation and cognition (choice C is wrong). The James-Lange theory asserts that emotional experience is primarily based on physiological arousal, and that each different physiological state is associated with an emotion (choice D is wrong).

Interactionist theories often use the dramaturgical approach to understand the processes behind social interactions. The dramaturgical perspective of health care might argue that: I.medical professionals use impression management to manage their presentations and influence the perceptions of others, such as their colleagues. II.medical professionals might dress in business suits while treating patients in a private practice setting, which is an example of the front stage. III.medical professionals might dress in casual clothing while associating with friends in an informal restaurant setting, which is an example of the back stage. A) I only B) I and II only C) II and III only D) I, II, and III

B. The dramaturgical sociological perspective argues that human interactions, and the presentation of the self, is situational and dependent on the audience, place, and time, similar to the presentations offered in theatrical performances. These presentations are governed through cultural expectations specific to the situation, and thus identities are not stable. Item I is true: The dramaturgical perspective suggests the use of impression management in most situations, including the specific example of health care settings (choice C can be eliminated). Impression management, also referred to as self-presentation, is the conscious or unconscious process of managing one's public image, as described above. For example, medical professions might use impression management at the hospital to present themselves as knowledgeable health care providers who are active in the treatment of patients (choice C can be eliminated). Item II is true: During social interactions in the front stage, people used impression management to choose their method of presentation (e.g., dress and behavior). Medical professionals dressing in a specific fashion to interact with patients is an example of front stage interaction (choice A can be eliminated). In the case described, the decision to dress in business suits, or other professional dress, might be made to reflect the provider's respect for their patients. Item III is false: During social interactions in the back stage, people are able to act as themselves without the worries of impression management. However, medical professionals dressing in a specific fashion to interact with friends is also an example of front stage interaction (choice D can be eliminated and choice B is correct). In the case described, the decision to dress in casual clothing might be made to reflect their desire to be comfortable with friends or dress in a manner that reflects the informal setting. The dramaturgical perspective would argue that most social interactions require some form of impression management. The back stage, then, might be represented through one's dress and behavior at home, either alone or in the presence of close relations.

Research indicates that professional burnout can result in thoughts of suicide. Suicidal ideation is most often associated with a psychological disorder in which broad category? A) Anxiety disorders B) Mood disorders C) Personality disorders D) Psychotic disorders

B. Thoughts of death or suicide are often associated with major depressive disorder, an example of a mood disorder (choice B is correct). Mood disorders are a group of diagnoses involving disturbances in mood or affect; these disorders describe a longitudinal emotional state. Anxiety disorders are a group of diagnoses involving excessive worries and include generalized anxiety disorder, panic disorder, and obsessive-compulsive disorder (choice A is wrong). Research does not show a significant association between these disorders and suicidal ideation, with the exception of post-traumatic stress disorder. Personality disorders are a group of diagnoses involving enduring maladaptive patterns of behavior and cognition and include antisocial personality disorder, avoidant personality disorder, and borderline personality disorder (choice C is wrong). Psychotic disorders are a group of diagnoses involving a loss of contact with reality and include schizophrenia and delusional disorder (choice D is wrong).

According to functionalist theorists, which of the following are manifest functions of the health care system in the United States? I. To maintain and improve the health of people so that all citizens are able to fulfill their normal social responsibilities, thus contributing to social continuity II. To educate patients on the health risks associated with harmful behaviors, such as the use of carcinogenic substances, thus helping to define normal and deviant behaviors III. To create medical institutions, such as hospital systems, which stand as centers of community values and identites, thus serving to maintain social stability A) I only B) I and II only C) II and III only D) I, II, and III

B. The structural functionalist perspective focuses on the contributions of different structures to overall social functions; theorists are concerned with how social structures interact to sustain social functioning. Manifest functions, in particular, are the intended contributions of social structures. Thus, this question is concerned with the purposeful functions of medical institutions, those objectives which the health care system is most concerned with accomplishing and those which are expected to be included in the public statements of all providers. Thus, according to the functionalist perspective, the manifest functions are expected to include the foundational questions of health care, addressing health maintenance and disease treatment. Item I is true: The most fundamental purpose of medicine, as a social institution, is to address the health of individuals (choice C can be eliminated). Furthermore, the item addresses concerns of structural functionalism, such as social norms and continuity. Item II is true: The education of patients, rather than the population, is also a core value in medicine (choice A can be eliminated). The physician-patient relationship serves to address health issues, including exposure to harmful substances. Furthermore, the item addresses concerns of structural functionalism, such as social norms, and the definition of deviance as those behaviors which do not fit the social norm. Item III is false: The primary purpose of medical institutions is to address concerns of health and illness (choice D can be eliminated and choice B is the correct answer). However, it is possible that the establishment of hospitals, and the subsequent values integrated into its environment, reflects the norms of the community. This result is better described as a latent function; latent functions are the unintended contributions of social structures. In the case of medicine, latent functions are those contributions which the social institution was not officially designed to offer. Furthermore, the item also addresses concerns of symbolic interactions, such as the hospital serving as a symbol of the community's values and identity.

According to functionalist theorists, which of the following are manifest functions of the health care system in the United States? To maintain and improve the health of people so that all citizens are able to fulfill their normal social responsibilities, thus contributing to social continuity To educate patients on the health risks associated with harmful behaviors, such as the use of carcinogenic substances, thus helping to define normal and deviant behaviors To create medical institutions, such as hospital systems, which stand as centers of community values and identites, thus serving to maintain social stability A) I only B) I and II only C) II and III only D) I, II, and III

B. The structural functionalist perspective focuses on the contributions of different structures to overall social functions; theorists are concerned with how social structures interact to sustain social functioning. Manifest functions, in particular, are the intended contributions of social structures. Thus, this question is concerned with the purposeful functions of medical institutions, those objectives which the health care system is most concerned with accomplishing and those which are expected to be included in the public statements of all providers. Thus, according to the functionalist perspective, the manifest functions are expected to include the foundational questions of health care, addressing health maintenance and disease treatment. Item I is true: The most fundamental purpose of medicine, as a social institution, is to address the health of individuals (choice C can be eliminated). Furthermore, the item addresses concerns of structural functionalism, such as social norms and continuity. Item II is true: The education of patients, rather than the population, is also a core value in medicine (choice A can be eliminated). The physician-patient relationship serves to address health issues, including exposure to harmful substances. Furthermore, the item addresses concerns of structural functionalism, such as social norms, and the definition of deviance as those behaviors which do not fit the social norm. Item III is false: The primary purpose of medical institutions is to address concerns of health and illness (choice D can be eliminated and choice B is the correct answer). However, it is possible that the establishment of hospitals, and the subsequent values integrated into its environment, reflects the norms of the community. This result is better described as a latent function; latent functions are the unintended contributions of social structures. In the case of medicine, latent functions are those contributions which the social institution was not officially designed to offer. Furthermore, the item also addresses concerns of symbolic interactions, such as the hospital serving as a symbol of the community's values and identity.

If the children in the experiment had been offered no material incentive (e.g., reward chips), how might this have affected the results? A) The children would not have given up on the harder problems. B) The experimental results may have been confounded by motivational variables. C) The children with unsolvable problems would have given up less frequently. D) Children with an internal locus of control would have found the matching patterns less intrinsically rewarding.

B. The students were given chips to redeem for toys, presumably, to ensure that the students put effort into the matching pattern tasks. Without providing some sort of an incentive to the children, it is possible that the experimental results may have been confounded by motivational variables—particularly, the children may have been far less motivated, and would have potentially exerted less effort, especially on the unsolvable pattern tasks (choice B is correct). Without a reward, the children would be more likely to give up on the harder (unsolvable pattern) tasks more readily (choices A and C are wrong). While the second paragraph states that "those on the extreme exogenous end of the scale usually need a long series of major obstacles before they give up," there is no indication that children with an internal locus of control would be less likely to have found the matching patterns less intrinsically rewarding (choice D is wrong).

A reasonable explanation of the researchers' hypothesis is that: A) those who view intelligence as a static trait will not attribute comprehension difficulties to the complexity of the text. B) difficult texts prompt incremental theorists to make attributions in a manner that is inconsistent with the self serving bias. C) incremental theorists are likely to possess better comprehension skills. D) the actor-observer effect is easily observed in comparisons between entity and incremental theorists.

B. This question requires an understanding of incremental and entity theories of intelligence. Incremental theorists believe that intelligence is malleable and expandable, while, as expressed in the passage, entity theorists believe intelligence to be static. A corollary of this debate is the role of effort. According to entity theorists, intelligence cannot be expanded through effort, while, according to incremental theorists, it can. If so, these two groups may make different interpretations about the extra effort required by a difficult comprehension task. Entity theorists would interpret extra effort as an indication that they are nearing the limit of their ability and would thus feel less confident about their understanding of the text. This interpretation is consistent with the principle of what social psychologists call the self-serving bias. Through the self-serving bias, people typically attribute negative outcomes to uncontrollable circumstances rather than to personal decisions or traits. Similarly, entity theorists attribute their comprehension difficulty to their static level of intelligence and the difficulty of the passage rather than to insufficient effort (choice A is wrong). Incremental theorists, however, would interpret extra effort as an effective means of expanding the limits of their intelligence. In this case, they would attribute comprehension difficulty to insufficient effort and would therefore interpret increased effort as a positive response to the difficulty posed by the text. This attribution is inconsistent with the self-serving bias because it attributes a negative outcome (comprehension difficulty) to factors within the person's control (insufficient effort). Accordingly, the researchers posited that incremental theorists' perceived comprehension would remain stable across text conditions (choice B is correct). The researchers' hypothesis pertains to perceived comprehension, not actual comprehension (choice C is wrong). The actor-observer effect refers to people's tendency to attribute other people's successes to circumstance and their failures to personal decisions or traits. The study in this passage does not pertain to people's perceptions of other people's performance or failure (choice D is wrong).

The episodic buffer allowed participants to: A) remember the instructions of the OLG task. B) form a visual picture of the video clip while recording response options. C) apply experience learned in prior games in order to generate options. D) avoid confusing the contents of one clip with those of another.

C. According to Baddeley, the working memory system is composed of four parts. The visuospatial sketchpad is a short-term visual store, the phonological loop is a short-term auditory store, the central executive is responsible for task-switching, and the episodic buffer integrates information from the other three systems as well as from long-term memory. An example of the contribution of the episodic buffer, therefore, is the recollection and application of prior game experience (choice C is correct). It is not directly responsible for the internal organization of experience (choice D is wrong). Rehearsal of verbal instructions would be the province of the phonological loop (choice A is wrong), while short-term visual memory of the video clips would be the province of the visuospatial sketchpad (choice B is wrong).

The Building Strong Families (BSF) Program is a part of a national initiative funded by the United States Department of Health and Human Services. BSF was developed to better understand the effect of well-designed interventions targeted at improving the relationships between romantic partners. Premarital interventions focused on measures associated with marital stability and satisfaction. Free services were provided to unmarried low-income couples. The program also offered financial incentives and additional supports (e.g., childcare, transportation, and meals). However, only 55% of enrolled participants attended at least one session. Models of health behavior might offer which potential explanation for this result? A) Professional support in strengthening one's relationship can paradoxically lead to external stressors, such as financial stressors, which can cause relationship strain. B) Preventative measures are used most often when it is believed that the public is susceptible to the problem, regardless of personal experiences. C) Premarital counseling is pursued in the specific context of perceived risk; it is often seen as a method of correction rather than an opportunity to strengthen successful relationships. D) Programs targeting specific populations, such as low-income couples, can be a stressful reminder of widespread social inequalities and outcomes.

C. Based on the information provided in the question stem, the program offered many services to help couples overcome barriers to receiving professional counseling, as well financial incentives. For this reason, it appears that participant behavior (e.g., the low attendance rate) was the result of factors internal to the relationship rather than issues with the program. For example, it is possible that some enrolled participants considered their relationship strong and therefore did not pursue the services offered (choice C is correct). Models of health behavior suggest that people often pursue professional interventions as the result of a perceived threat, as opposed to as a preventative step. Furthermore, this conclusion is supported in the final paragraph of the passage, which discusses the role of cognitive biases in pursuing professional services such as premarital counseling. It is possible that the decision to pursue professional services could cause stress; however, this explanation is less applicable to the BSF program, which offered many services to support participants (choice A is wrong). The results of the BSF program do not support the claim that people pursue services when the public is believed to be at risk for a certain problem; for example, the fact that divorce rates are often cited does not appear to have had an effect on the utilization of professional services (choice B is wrong). Finally, while it is possible that the decision of BSF to target low-income couples contributed to the low attendance rates, this conclusion is not supported in the passage (choice D is wrong).

Sociologists refer to women's role as family caregivers in addition to their role as professionals as the "second shift." How might the "second shift" be interpreted as a psychosocial stressor? A) It exacerbates women's sense of self-efficacy in the workplace. B) It results from prejudice against working women. C) It may lead to identity interference. D) It is a function of the glass ceiling, which leads to a lower standard of living.

C. By definition, women's "second shift" refers to an ongoing duality of roles (the role of working woman and the role of working professional and the role of family caregiver). When the demands of these roles, such as time and energy, conflict, the resulting tension is called identity interference. Identity interference is the principle that people experience stress when two aspects of their self-identity are in conflict. Thus, the "second shift" could be understood as a cause of identity interference, which then constitutes a psychosocial stressor (choice C is correct). It is not the result of prejudice (choice B is wrong). Self-efficacy is Albert Bandura's term for one's feeling capable of accomplishing a given task; compounding one's roles would not increase self-efficacy nor would it likely cause stress (choice A is wrong). The glass ceiling refers to unofficial limits on women's advancement in the workplace and does not directly relate to their additional roles outside the workplace (choice D is wrong).

Because this study factored in perceptions of support as well as actual support, if both factors were found to be equally as valuable in increasing the odds of survival we might say this is an example of what psychological principle? A) Social perception B) Attachment theory C) Causal attribution D) Social intelligence

C. Causal attribution describes how a perceiver takes information and ascribes causes. Thus, a person might attribute his or her well-being to social support, real or perceived (choice C is correct). Social perception describes the processes by which a person comes to understand how he or she is in the world (choice A is wrong). Attachment theory describes the relationship between the child and the primary caregiver (choice B is wrong). Social intelligence refers specifically to how well one relates to others and to one's tasks in the world (choice D is wrong).

Suppose Sarah is kneading a piece of clay in her hands. Which of the following is NOT a strategy relevant to conservation? A) Sarah realizes the clay is the same piece of clay, regardless of its shape. B) Sarah notices that she can shape the clay into a ball, then a pancake, then back into a ball again. C) Sarah knows that the clay still exists even if her mother puts it back into its container. D) Sarah discovers that squeezing the clay in the middle forces it to become bulkier on the sides.

C. Conservation is the term Piaget used to describe children's recognition of constancy and consistency in physical matter despite changes in shape or container. This awareness is characteristic of the stage of concrete operational thought, which is usually achieved between the ages of 6 or 7 and 11 or 12. Conservation can be realized through three insights, including identity, reversibility, and reciprocity. In identity, the child recognizes that the physical object is the same object, regardless of how it is manipulated (choice A is true and therefore wrong). In reversibility, the child recognizes that a given manipulation can be reversed to give the object its initial appearance (choice B is true and therefore wrong). Finally, in reciprocity, the child realizes that a manipulation of one dimension or aspect yields a corresponding change in another aspect. As a result, the manipulation is understood to change the original object rather than create a new one (choice D is true and therefore wrong). The realization that an object continues to exist despite its disappearance from the visual field is known as object permanence (choice C is NOT directly related to conservation and is therefore correct).

The Big Five personality model consists of what five traits? A) Awareness, cognitive ability, extraversion, agreeableness, and neuroticism B) Openness, cognitive ability, extraversion, altruism, and neuroticism C) Openness, conscientiousness, extraversion, agreeableness, and neuroticism D) Awareness, conscientiousness, extraversion, altruism, and neuroticism

C. The Big Five personality model includes openness, conscientiousness, extraversion, agreeableness, and neuroticism; the first letters of each spell OCEAN, a common mnemonic for remembering these (choice C is correct). Cognitive ability is a mental ability, not a component of personality (choices A and B can be eliminated without actually knowing the five-factor model). Although altruism and agreeableness are related, it is agreeableness that is part of the five-factor model. Furthermore, awareness is a vague word that is also more frequently associated with cognitive abilities rather than personality (choice D is wrong).

Theorists using the conflict perspective to address questions of health, illness, and medicine are LEAST expected to consider: A) the commodification of health care delivery. B) the presence of health and health care disparities. C) the power of professionals in patient-provider relationships. D) the dependence of medical institutions on profit.

C. Each of the main theoretical perspectives—structural functionalism, conflict theory, and symbolic interactionism—is expected to approach experiences of health and health care differently. The conflict theorist approach considers the competition for limited resources. According to this theory, limited resources, a negative result of capitalism, cause inequalities and power differentials in society. This understanding can be extended to the medical institution. Furthermore, Karl Marx, the founding conflict theorist, focused on the internal tensions produced in capitalist societies. Thus, it is expected that the conflict perspective would consider the commodification of health care and the dependence of institutions on profit (choices A and D can be eliminated). Health care can thus be purchased and sold in a marketplace, which allows those with resources and influence to control decisions about health care, thus maintaining inequalities. Furthermore, these social inequalities determine the distribution of health care access and resources. Thus, it is expected that the conflict perspective would consider subsequent disparities in health and health care (choice B can be eliminated). However, while conflict theorists are interested in the creation and maintenance of power differentials, as proponents of a macro-sociological perspective, this interest is applied to social structure as a whole. The patient-provider relationship would perhaps be of more interest to symbolic interactionists, as symbolic interactionism is a micro-level perspective focused on the transfer of information through communication between individuals (choice C is not an expected focus of conflict theorists and is the correct answer). Furthermore, it is possible that a power differential is inherent in the provider-patient relationship as a result of the expertise necessitated in the education and training of medical professionals, thus making an egalitarian relationship difficult to create

As discussed in the passage, donation preferences are shaped through knowledge, incentives, and confidence. The idea of incentives suggests which of the following? I. Parents are more likely to donate to causes such as education. II. Older individuals are more likely to donate to causes such as healthcare. III. African Americans are less likely to donate to causes such as civil rights. A) I only B) II only C) I and II only D) I, II, and III

C. In paragraph 1, it is explained that people tend to donate to causes within the "sphere of the individual" as this offers the best incentive. Item I is true: Education is a cause psychologically close to parents; thus, it is expected that parents would be more likely to donate to related organizations (choice B can be eliminated). Item II is true: Health care is a cause that grows increasingly psychologically close as a person ages; thus, it is expected that older individuals would be more likely to donate to related organizations (choice A can be eliminated). Item III is false: Civil rights is a cause psychologically close to African Americans; thus, it is expected that African Americans would be more, not less, likely to donate to related organizations (choice D can be eliminated and choice C is the correct answer).

NPM-2005 did not account for the personal experiences of the reported families. This is a suggestion for future directions in research. Researchers predict that the aversive impact of stressful life events might explain the elevated rates of some forms of child maltreatment in certain groups. This effect is expected to be the most pronounced in: A) native families. B) traditional immigrant families. C) non-traditional immigrant families. D) all immigrant families.

C. It is true that all families, both native and immigrant families, experience some stressful life events. However, this stress is expected to be highest in non-traditional immigrant families, which are defined as refugees (choice C is correct). It is not expected that native families will tend to be impacted by stressful life events at a greater rate than immigrant families (choice A is wrong). In contrast to traditional immigrant families, which tend to emigrate from former Dutch colonies, non-traditional immigrant families are more heterogeneous in terms of their ethnicities. However, the fact that these groups migrated due to persecution suggests a shared experience that might contribute to traumatization (choices B and D are wrong). Furthermore, the complicated process of gaining political asylum also suggests more post-migration stress. This is significant when considering rates of child maltreatment. For example, this stress could lead to the development of parental disorders, like post-traumatic stress disorder (PTSD). This involves three clusters of symptoms, including physiological hyperarousal, which might include angry outbursts.

There are multiple sociological perspectives on deviance. Which of the theorists below are expected to consider the following questions about non-normative behaviors? Differential association theorists; "How can people resist deviance? Labeling theorists; "Who defines deviance?" Structural strain theorists; "How do norms affect deviance?" A) III only B) I and II only C) II and III only D) I, II, and III

C. Item I is false: Differential association argues that deviant behaviors are learned through interactions between individuals and their communities. These theorists contend that people become deviant when their contacts have an excess of definitions favorable to deviant behaviors. "How can people resist deviance?" is therefore not an expected question (choices B and D can be eliminated). The main criticism of differential association is that it reduces individuals to their environment. Because it states that those in deviant communities learn to be deviant themselves, the possibility of resistance is not considered. Item II is true: Labeling theory argues that deviant behaviors are seen as the result of social processes of labeling. "Who defines deviance" is therefore an expected question (choice A can be eliminated). Labeling theorists do indeed address this concern; for example, these theorists are interested in the mechanisms through which power contributes to the construction of deviance (e.g., agents of social control). Item III is true: Structural strain theory argues that deviant behaviors are the result of tension between the accepted social goals and the institutionalized means available to achieve these goals. "How do norms affect deviance?" is therefore an expected question (choice C is correct). Structural strain theorists consider the effects that social norms have on our behaviors, normative or not, and suggest that there is pressure to use deviance when the social structure does not support achieving the accepted goals.

Knowledge of specific olfactory thresholds for various substances would be most useful to which of the following clinicians? A) An eating disorders counselor trying to rule out a diagnosis of anorexia nervosa B) A psychologist trying to determine if a patient has a cocaine addiction C) A psychiatrist assessing for positive symptoms of schizophrenia D) A therapist treating elderly individuals with delirium

C. Positive symptoms of schizophrenia include delusions and hallucinations. While hallucinations typically occur in either the visual or auditory senses, they can occur in any of the five senses, including the olfactory sense. Thus, by knowing the detection threshold for various substances, a clinician could determine if a patient's claim that he/she smelled a particular substance was a function of hallucinations or an accurate olfactory detection (choice C is correct). While cocaine can be snorted, the ability of a patient to actually detect the odor of cocaine would be irrelevant to their addiction (choice B is wrong). Olfactory processes are not associated with anorexia nervosa or delirium (choices A and D are wrong).

Conflict theorists are concerned with stratification in societies. The argument least consistent with this perspective is which of the following? A) Stratification is the result of competition between different social classes. B) Stratification is the result of the exploitation of subordinate groups. C) Stratification is the result of the need for the maintenance of social order. D) Stratification is the result of capitalist motivation to accumulate wealth

C. The conflict perspective argues that social structures, including structured social stratification, reflect the competition for limited resources (choice A is consistent and can be eliminated). These structural inequalities happen because those with the most power (the dominant groups) maintain their positions of power through the suppression of those with the least power (the subordinate groups; choice B is consistent and can be eliminated). The traditional conflict theorists found this to be of particular interest in capitalist societies; Marx, for example, argued that capitalism produced internal tensions between those who controlled the means of production and those who labored the production (choice D is consistent and can be eliminated). However, the argument that stratification contributes to social order is not expected to be made from the conflict perspective; this is better described as a functionalist argument (choice C is not consistent and is the correct answer). For example, Marx argued that the tensions associated with capitalism would lead to its self-destruction, suggesting that he did not find pronounced stratification to contribute to social order.

The reported findings for Miami, Florida might be explained through: A) decreases in the racist policies that contribute to racial segregation. B) decreases in the economic progress of minorities in the state of Florida. C) increases in the rate of migration to the United States. D) increases in the efforts to decrease residential segregation in central cities.

C. The passage states that Miami, Florida is one of the cities that experienced increases in residential segregation as studied using the data from the past two census reports. Therefore, the correct answer should reflect a change that is expected to cause increases in racial disparities. The rate of migration in the United States has had profound effects on the demographic composition of the nation's largest metropolitan areas. Recent immigrants to the United States are most often Hispanic, and this is heightened in Miami, which has a Hispanic population share of 66 percent based on the most recent census data. It is also known that immigrants tend to be located in ethnic communities ("ethnic enclaves"), which suggests residential segregation (choice C is correct). The decrease in racist policies responsible for the historic concentration of minorities in poor neighborhoods explains much of the decline in residential segregation nation-wide; it therefore does not explain the observed incline in Miami (choice A is wrong). The increase in formal policies prohibiting racism in the United States, one of the modern social changes of importance for the MCAT, has contributed to the economic progress (not regression) of minorities (choice B is wrong). It has been noted, for example, that the rise of the black middle-class has contributed to the decline in residential segregation; this can be attributed, in part, to the migration of these black Americans to the suburbs. The increase in efforts to decrease residential segregation, though debatable, suggests declines in racial segregation; it therefore does not explain the observed incline in Miami (choice D is wrong).

All of the following are possible consequences of labeling deviance, EXCEPT: A) Retrospective labeling B) Stigma C) Differential association D) Medicalization of difference

C. The premise of labeling theories of deviance states that once a norm violation has been observed and labeled deviant, several long-term consequences occur that involve the identification and differential treatment of the individual whose behavior is labeled deviant. The correct choice will describe a relationship between an individual and society in a way that does not involve social surveillance or ostracization. Differential association describes processes through which individuals tend toward conformity or deviance based on social encouragement, so it does not constitute a consequence of labeling deviance (choice C is correct). Retrospective labeling is a consequence of being labeled deviant because it means interpreting someone's past behavior in terms of a current act of deviance (so choice A can be eliminated). Being labeled deviant also leads to acquiring a stigma, which is a negative label that alters self-concept and leads to social isolation (choice B can be eliminated because it is a consequence of labeling deviance). The label ascribed to someone who exhibits morally or legally deviant behavior can also be transformed into a medical condition, which further stigmatizes the individual (choice D can be eliminated because it is a consequence of labeling deviance).

Access to and utilization of preventative care is often evaluated through a community's use of prophylactics, defined as medical or public health procedures that prevent disease. Immunization is a common and widespread example of prophylaxis. Additional examples of prophylaxis would be expected to include all of the following, EXCEPT: A) birth control methods, including condoms. B) regular moderate physical activity. C) cesarean section (c-section) delivery. D) fluoride therapy and tooth cleaning.

C. The question describes prophylaxis as a method of preventative care. Birth control can be used as a preventative measure, often to prevent pregnancy (choice A is an example of prophylaxis and can be eliminated). Furthermore, considering the example provided, condoms can also be used to prevent sexually transmitted disease. Physical activity has numerous health benefits, including the ability to reduce one's risks for diseases such as heart diseases, muscular and skeletal diseases, and even some forms of cancer (choice B is an example of prophylaxis and can be eliminated). Cesarean section, although a common procedure, is a surgical method; it is performed when physicians deem it a safer method of delivery, but is generally not used preventatively, but rather as a response to complications, especially in the first instance (choice C is not an example of prophylaxis and is the correct answer). Fluoride therapy and tooth cleaning are dental forms of prevention, helping to prevent dental disease, such as gingivitis, which could have a negative effect on overall health (choice D is an example of prophylaxis and can be eliminated).

Non-traditional immigrant families are overrepresented in reports of all of the following, EXCEPT: A) emotional abuse. B) physical abuse. C) sexual abuse. D) other abuse.

C. The results are presented in Figure 1. It is important to note that non-traditional immigrants accounted for 12 percent of the participant sample (paragraph 4). Reports of child maltreatment are higher than expected based on this representation in the sample in the following categories: emotional abuse (25 percent), physical abuse (17), and other abuse (16; choices A, B, and D can be eliminated because non-traditional immigrant families are overrepresented in these reports). However, non-traditional immigrants account for a mere six percent of reports of sexual abuse, which is lower than expected based on their representation in the sample (choice C is correct as non-traditional immigrant families are underrepresented here).

The current research measured exposure to stress through a questionnaire focused on significant life changes. However, other forms of stress can also have an impact on marital outcomes. Which of the following correctly pairs the type of stressor with a scenario describing that stressor? A) Catastrophes; sudden divorce filing B) Significant life changes; wartime displacement C) Daily hassles; mortgage deadlines D) Ambient stressors; moving homes

C. There are three main forms of stressors: catastrophes, significant life changes, and daily hassles. Catastrophes are unpredictable, large-scale stressors, such as natural disasters, that impact many people to a significant extent. A sudden divorce filing is an example of a significant life change, not a catastrophe (choice A is wrong). Significant life changes are high stress events that occur with greater frequency than catastrophes, such as marriage, divorce, or loss of a loved one. Wartime displacement is an example of a catastrophe, not a significant life change (choice B is wrong). Daily hassles are common irritations in life, such as scheduling errors. A mortgage deadlines is an example of a daily hassle as it impacts individuals regularly (choice C is correct). Ambient stressors are caused by one's environment, such as noise. Ambient stressors can be related to daily hassles, such as in the case of pollution. However, moving homes is an example of a significant life change, not an ambient stressor (choice D is wrong).

Different methods of social stratification can create multiple hierarchical threats for certain individuals. The concept of triple jeopardy describes the specific situation in which a person's different social statuses interact with one another so that the negative effects of each social status contribute to negative social experiences and potential poorer qualities of life. This population is expected to include: A) black men with an average age of 25. B) white men with an average age of 75. C) black women with an average age of 75. D) white women with an average age of 25.

C. This question requires an understanding of social trends of discrimination as a result of age, race, and sex to determine which individuals are expected to have the lowest statuses. Modern societies often discriminate against older individuals, black individuals, and women. These ageist, racist, and sexist attitudes are expected to pose to greatest threat ("triple jeopardy") to older black women (choice C is correct). The average black man who is 25 experiences a single form of discrimination (racism) and can still benefit from his other statuses (choice A is wrong). The average white man who is 75 experiences a single form of discrimination (ageism) and can still benefit from his other statuses (choice B is wrong). The average white woman who is 25 experiences a single form of discrimination (sexism) and can still benefit for her other statuses (choice D is wrong).

Many forces influence how people relate to one another, become socialized beings, and learn to develop relationships. Which of the following is NOT one of the agents of socialization? A) Family B) School C) Travel D) Television

C. While travel can be highly educational, introducing people to other cultures and ways of life, it is not specifically an agent of socialization, helping people to learn how to relate, as one can travel alone and avoid contact with others (choice C is correct). Family is one of the first and primary sources of socialization in which a person learns how to relate to others—one's parents, siblings, and extended family, depending on family structure. The lessons learned here often form the bases for many future relationship choices (choice A is wrong). School is another primary source of socialization, as teachers try to educate children about the values of the specific culture and the important skills needed to survive in that culture (choice B is wrong). Television, along with other forms of mass media, provide people with a glimpse of how to relate to the larger world, introducing people to other lifestyles as well as popular trends and ideas (choice D is wrong).

The Implicit Association Test (IAT) was developed at Harvard to measure unconscious biases that people may have. The subject is presented with words or photos, one at a time, and asked to categorize the stimuli as either "good" or "bad" via the computer keyboard. The associations between the stimulus and the category of good or bad are measured via the response time; when the requested categorization (i.e. a particular face with distinctive racial features to be categorized with "good") is congruent with the subject's pre-existing bias (if any) the subject responds faster than when the associations do not match. The discrepancies in response time indicate the bias a person has; longer response times indicate more difficulty in matching the pairings. What might the IAT of a bi-cultural Asian American with a low BII show if she were asked to pair the word "self" with either American or Asian? A) She would show a bias during American/self pairings. B) She would not show any bias. C) It would depend on the context that was primed prior to her beginning the IAT. D) She would show a bias during Asian/self pairings.

C. Without knowing what priming condition the respondent had been exposed to prior to taking the IAT, it is impossible to assess what the subject's performance would likely be. Had the person been exposed to an American priming condition prior to the IAT, she would likely be faster with pairing the self with the Asian prime, and vice versa, as those with low BII tend to have the opposite cultural identity primed when presented with a cultural prime (choice C is correct). Since we do not know what the priming condition was, we cannot tell if she would have a bias during the American/self or Asian/self pairings (choices A and D are wrong). Since she has a low BII we can know that she would show a bias, but that the bias would differ depending on which cultural identity was primed before the IAT (choice B is wrong).

It has been proposed that there may be variations in the OXTR gene for individuals with and without a specific psychological disorder. In particular, researchers cite the OXTR gene's role in a variety of social behaviors, including trust, social comprehension, and attachment as being evidence that it may be different between people with and without this disorder. It is likely that researchers think that variation in the OXTR gene contributes most to which of the following disorders? A) Schizophrenia B) Obsessive-compulsive disorder C) Attention deficit/hyperactivity disorder D) Autism spectrum disorders

D. Although research does suggest that OXTR gene may be associated with general psychopathology, autism spectrum disorders are most characterized by deficits in social behaviors such as trust, social comprehension, and attachment. Research has suggested that autistic individuals may be more likely to exhibit an A allele (choice D is correct). Schizophrenia is often characterized by delusional thinking, hallucinations, and disorganized behavior, but social deficits are often only a byproduct of other psychotic symptoms (choice A is wrong). Obsessive-compulsive disorder is characterized by persistent and obsessive thoughts and repetitive behaviors, but is also not strongly linked to social deficits (choice B is wrong). Attention deficit/hyperactivity disorder (ADHD) is characterized by problems with maintaining attention and controlling impulsive behaviors, but is only tangentially linked to social deficits (choice C is wrong).

Which of the following reflects the researchers' attempt to minimize adaptation effects during the experiment? A) Using wild mice rather than domesticated mice B) Using sugar water rather than pond water during the training stage C) Rearranging the bottles after a human subject detected sulfur D) Limiting the human subjects' inhalation time to two seconds

D. An adaptation effect is the phenomenon by which an organism becomes accustomed to a sensory stimulus, which results in a higher threshold necessary for detection of the stimulus. Since the experiment in this study was designed to measure detection thresholds, it was necessary to limit participants' inhalation time to two seconds so that they would not become accustomed to the smell of sulfur. Otherwise, their olfactory thresholds for the detection of sulfur may have measured higher than their actual levels (choice D is correct; choices A, B, and C are wrong).

According to Piaget's theory of development, at which stage do children learn that things can be represented through words and images? A) Formal operational stage B) Sensorimotor stage C) Concrete operational stage D) Preoperational thought stage

D. During Piaget's preoperational thought stage (around ages 2 to 7), children learn that things can be represented through symbols, such as words and images; this is an important foundation for the development of more advanced language skills. At this stage they still lack logical reasoning but they begin to understand the world of representational symbols, so it is during the preoperational thought stage where children learn that things can be represented through words and images (choice D is correct). The formal operational stage occurs when a child reaches the age of 12 and above. This is where the child begins to understand complex abstract thought and reasoning, as well as morality (choice A is wrong). The sensorimotor stage occurs during the first two years of life where a child explores his or her world with the five senses and through movement. This is also where they learn object permanence (choice B is wrong). The concrete operation stage is from ages 7 to 12, where the important lesson is conservation—that quantities remain the same even if one changes their shape. This is also the age where children begin to grasp abstract mathematical concepts (choice C is wrong).

A contemporary of Freud's, Carl Rogers was largely responsible for developing Humanistic Psychology. Rogerian psychology, while stemming from psychoanalysis, and considered to be psychodynamic, is different from Freudian theory because: A) Freud believed that humans were motivated by unconscious anxieties and desires, whereas Carl Rogers did not believe in the unconscious. B) Freudian theory emphasizes social influences, while Rogerian theory focuses on biological impulses. C) Freudian theory suggests that people were motivated by the need to become self-actualized whereas Rogers posited that people were motivated to avoid anxiety. D) Freudian theory emphasizes innate biological drives, and focuses on the conflict between the individual and societal demands, and Rogerian theory focuses on the individual's experience within the social environment, and posits that individuals are motivated to become self-actualized.

D. Freud posited the existence of the Id, which contains the biological impulses and unconscious desires and emphasized that the Superego and the Ego formed in order to manage the individual's interactions with society. Carl Rogers regularly recognized the importance of the interaction of the individual within society, and posited that people were motivated by the desire to become actualized as the best possible versions of themselves (choice B is correct). While Freud did believe that humans experience unconscious anxieties and desires, so did Carl Rogers. This can be inferred from the question stem: psychodynamic theories all recognize the influence of the unconscious (choice A is wrong). Freudian theory emphasizes biological, or innate, impulses (Id), and Carl Rogers did not primarily emphasize biology (choice B is wrong). Rogerian theory (not Freudian theory) suggests that people were motivated by the need to become self-actualized, and Freud (not Rogers) posited that people were motivated to avoid anxiety (choice C is wrong).

Given the information presented in Table 1, the data shown in Figure 1 suggest that: A) a mammal's relative olfactory bulb size is a reliable predictor of olfactory sensitivity to sulfur-containing compounds, but absolute size is not. B) a mammal's absolute olfactory bulb size is a reliable predictor of olfactory sensitivity to sulfur-containing compounds, but relative size is not. C) both absolute and relative olfactory bulb size are reliable predictors of a mammal's olfactory sensitivity to sulfur-containing compounds. D) neither absolute nor relative olfactory bulb size are reliable predictors of a mammal's olfactory sensitivity to sulfur-containing compounds.

D. From Table 1, it is evident that the human olfactory bulb has a larger absolute size than the mouse olfactory bulb, while the mouse olfactory bulb has larger relative size. If absolute size was a reliable predictor of olfactory sensitivity to a given odorant, then the percentage of human subjects that detected sulfur at a given threshold should have exceeded the percentage of mouse subjects that detected sulfur at that threshold across all three odorant conditions. In contrast, if relative size was a reliable predictor, then the percentage of mouse subjects that detected sulfur at a given threshold should have exceeded the percentage of human subjects that detected sulfur at that threshold across all three odorant conditions. Based on this information alone, it would be impossible for both relative and absolute size to be predictors of olfactory sensitivity (choice C is wrong). From the data depicted in Figure 1, however, it is apparent that a greater percentage of mice detected sulfur at <5 ppt in the ferret and bobcat conditions, but a greater percentage of humans detected sulfur at the same threshold in the weasel condition. Thus, neither absolute nor relative olfactory bulb size was a reliable predictor of olfactory sensitivity (choice D is correct; choices A and B are wrong).

A scientist wants to test the heritability of sexual attractiveness in college-age females. This could be best accomplished by: A) using monozygotic twin studies to measure the percentage of sexual attractiveness caused by genes. B) using dizygotic twin studies to measure the percentage of sexual attractiveness caused by genes. C) using dizygotic twin studies to measure the percentage of variation in sexual attractiveness attributable to genetics. D) using monozygotic twin studies to measure the percentage of variation in sexual attractiveness attributable to genetics.

D. Heritability is best tested with monozygotic twin studies, since monozygotic twins share the same genes. Also, heritability is defined as the proportion of observable differences in phenotype attributable to genetic causes (choice D is correct). Heritability does not measure the percentage of an observable behavior caused by genes, but only has meaning within the context of proportion attributable to variation (choice A is wrong). The other two answer choices can be eliminated because heterozygotic twin studies do not measure heritability as well as monozygotic twin studies (choice B and choice C are wrong).

Suppose that the researchers who conducted the experiment described in the final paragraph concluded, based on the results, that while experienced detectives are only moderately better at lie detection than are untrained laypeople, the polygraph is in fact a highly effective tool for this purpose. Which one of the following, if true, would cast the most doubt on this conclusion? A) Unlike most people taking a polygraph examination, the interviewees in the experiment knew that they would suffer no serious or negative consequences arising from their lies. B) Compared to the average detective, those detectives who participated in the experiment were unusually astute and adept at lie detection. C) Because the interviewees who took the polygraph were instructed to lie for purposes of the experiment, they were apt to experience less guilt than if they were lying to protect their own interests. D) The laypeople demonstrated an extremely low level of success at lie detection.

D. If the experimenters concluded that the polygraph is a highly effective instrument for detecting lies based upon the fact that it was 21% better than the detectives (who were 17% better than the laypeople), it would be necessary to know what the results were for the laypeople. If the laypeople did an extremely poor job at lie detection (let's say they scored less than 10% accuracy at lie detection), then the polygraph results would still be no better than chance. The experimenters should have offered data indicating an overall success rate, not simply a relative one (choice D would cast doubt upon the experimenters' conclusion and is thus correct). If the interviewees who took the polygraph were less likely to experience the physiological effects of stress, whether because they knew there would be no adverse consequences or because they believed they really weren't "lying" per se, this would actually strengthen the experimenters' conclusion. If the polygraph could still be that much more effective at lie detection than trained professionals, even when physiological indicators of stress (which is what the test purports to measure), are low, then it would be all the more useful and valuable (choices A and C would not cast doubt upon the experimenters' conclusion and can be eliminated). Similarly, if the detectives who participated were uncommonly skilled at lie detection and the polygraph was still substantially better, this would strengthen the claim that the polygraph is very effective (choice B would not cast doubt upon the experimenters' conclusion and can be eliminated).

During the research period, 16% of wives with moderate forecasts and 12% of wives with positive forecasts divorced. Despite high divorce rates, many couples decided to forgo professional relationship counseling. This can be attributed to a number of factors, including the social stigma surrounding such services. Which of the following is true of stigma? I. It is applied to deviations in personal traits rather than external deformities. II. It is defined as disapproval resulting from actual or perceived social deviance. III. It can lead to demeaning labels that are applied to entire social groups. A) II only B) III only C) I and II only D) II and III only

D. Item I is false: Social stigma is a negative association with a person or group. This can result from the perception, correct or incorrect, of numerous characteristics, including personal traits, such as mental illness, or external deformations, such as visible manifestations of eating disorders (choice C can be eliminated). Item II is true: Deviance is the violation of social norms; this violation, in turn, is seen as unacceptable to the society at hand (choice B can be eliminated). Item III is true: Labels are often applied to individuals deviating from the social norm, thus perpetuating social stigma (choice A can be eliminated and choice D is the correct answer). For example, social stigma is a widespread problem in mental health; a demeaning label applied in this context might be "insane" or another related word.

Jax, a gay teenager, was kicked out of his parents' house when he came out as gay. Now he lives with a community of other teenagers in a youth center and he considers this community to be his family. Which of the following specific types of groups is this community for Jax? An in-group A primary group A reference group A) I only B) I and II only C) II and III only D) I, II, and III

D. Item I is true: an in-group is a group that an individual belongs to and feels him/herself to be a member of (choice C can be eliminated). Item II is true: a primary group serves expressive functions, such as meeting emotional needs. A family is an example of a primary group and Jax considers his teenage friends to be his family (choice A can be eliminated). Item III is true: a reference group is a group that one might compare oneself to based on the fact that all members share a common identity. Jax can compare himself to his teenage friends because they are all in a similar situation—being young and living primarily in shelters (choice D is correct; choice B can be eliminated).

Micro-sociologists are interested in the mechanisms through which individuals shape their social realities. Researchers are able to answer these questions using all of the following theoretical paradigms with an exclusive focus on micro-factors, EXCEPT: A) constructionism. B) interactionism. C) the dramaturgical perspective. D) the feminist perspective.

D. Micro-sociologists focus on the effects on individual interactions, like the social meanings created through these interactions. Each of the theories listed consider micro-level social factors. However, the feminist perspective is not limited to consideration of the individual (this perspective does not place an exclusive focus on micro-factors and choice D is the correct answer). This sociological approach studies the differences in the social experiences of men and women, thus considering micro-level questions, such as how gender is constructed through these interactions. The feminist perspective, however, is also concerned with the social structures that perpetuate gender inequalities in modern societies, such as the institution of marriage.

All of the following biological developmental milestones that change the brain occur during the adolescent years, EXCEPT: A) Cell proliferation (particularly in the limbic system and prefrontal lobes) B) Myelination C) Synaptic pruning (of unused or unnecessary connections) D) Codification of neural networks for basic motor skills

D. Neural network development for motor skills is part of early brain development and is essentially complete for basic motor skills (such as crawling, walking, jumping, writing, etc.) before adolescence (choice D is correct). Cell proliferation, particularly in the prefrontal lobes and in the limbic system, is one of the changes to the brain that occurs during adolescence, as the prefrontal cortex is required for abstract thinking (choice A is wrong). Myelination, the creation of myelin sheaths around neurons in the brain to strengthen connections, is another change during adolescence (choice B is wrong). Synaptic pruning, which clears the brain of unnecessary connections, is the third major developmental change in the brain during adolescence (choice C is wrong).

Which of the following is NOT a Cluster B personality disorder? A) Antisocial personality disorder B) Borderline personality disorder C) Histrionic personality disorder D) Paranoid personality disorder

D. Personality disorders are grouped based on similarities in their descriptions. Cluster B disorders are characterized by heightened emotional arousal as well as dramatic and/or erratic behavior. Paranoid personality disorder is characterized by the odd, paranoid behavior exhibited by an individual; therefore, it is a Cluster A disorder, NOT a Cluster B disorder (choice D is correct). Antisocial, borderline, and histrionic personality disorders are all characterized by emotional disregulation and/or overreaction (choices A, B, and C are wrong).

Later in her interview Subject 103 talked about her experience with smoking. She mentioned that she was interested in quitting, but that stopping smoking made her feel irritable and stressed. Smoking was the only thing that calmed her down, she told the interviewer. In this case, irritability is one sign of: A) psychological dependence. B) biofeedback. C) physiological arousal. D) physical dependence.

D. Physical dependence is evidenced by symptoms of withdrawal that are calmed by further use of the addictive substance. In this case, Subject 103 experiences irritability, but it is calmed by the nicotine she gets from smoking (choice D is correct). Psychological dependence can result from using a drug to cope with painful emotions. Although Subject 103 experiences painful emotions because she does not have a community that accepts her identity, the information in the question stem deals with her physical symptoms of dependence and not with the potential underlying psychological causes (choice A is wrong). Biofeedback is a therapeutic technique in which individuals are given information about their autonomic responses to stressful situations and taught to use this information to control their responses (choice B is wrong). Physiological arousal is an excitation of the body's internal state; it is an aspect of emotional response (choice D is wrong).

The researchers found that the more complex the social support, the stronger the association was between social support and increased odds for survival. As a result, which of the following supportive situations would be LEAST likely to increase one's odds for survival? A) Living in an assisted living community with access to an occupational therapist, psychotherapist, medical care, and recreational activities. B) Living in a nursing home, but with a constant stream of family and friends visiting and providing outings. C) Living alone with frequent visits to therapists, medical staff, and friends in the community. D) Living in one's own home with a supportive spouse.

D. Researchers found the binary factors—such as being married or living alone or not—were not sufficient indicators in themselves of a strong supportive network. Thus, living in one's own home, even with a supportive spouse, is the least likely to be a supportive environment (choice D is correct). While the patients in living in assisted living communities or nursing homes may not be considered ideal when compared to being at home, both have wide social networks with access to medical care (choices A and B are wrong). Although the patient may live at home alone, he or she still frequently interacts with a wider network in the community (choice C is wrong).

Robert Merton proposed the concept of unintended consequences, defined as unintended outcomes of purposeful social action. This suggests the importance of routinely evaluating models of social intervention, such as global health initiatives, for unintended consequences. For example, coercive vaccination caused resistance among individuals and communities during smallpox eradication efforts in India. In this case, the consequential drug resistance is best described as which of the following? A) A manifest function B) A latent function C) A manifest dysfunction D) A latent dysfunction

D. Robert Merton is the theorist credited for the distinction between manifest and latent functions and dysfunctions. In the global health situation described, the reported drug resistance among individuals and communities in India is an example of a latent consequence of social intervention (choices A and C are wrong). Latent functions and dysfunctions are the unrecognized, unintended consequences of social patterns; manifest functions and dysfunctions are the recognized, intended consequences of social patterns. Furthermore, this can be described as social dysfunction, rather than a function, because it is an undesirable consequence that can reduce social stability (choice B is wrong and choice D is correct). Thus, the two-by-two theory could be applied to choose the correct answer here.

If a bisexual female reports feeling social pressure to be either lesbian or straight, which causes her significant stress, what term best describes the stress this subject is feeling about her social role? A) Role conflict B) Master status C) Role exit D) Role strain

D. Role strain is when having a single status results in conflicting expectations; if a bisexual female reports feeling social pressure to be either lesbian or straight, which causes her significant stress, she is experiencing conflicting expectations about her bisexual identity (choice D is correct). Role conflict happens when there are conflicting societal expectations for multiple statuses held by the same person. For example, a male kindergarten teacher—societal expectations for being a man and being a kindergarten teacher easily come into conflict (choice A is wrong). Master status is when one part of someone's identity comes to dominate their identity, to the exclusion of other parts of their identity (choice B is wrong). Role exit is the process of disengaging from one role in order to take up another. An example is the process of disengaging from a "pre-med" identity to take on a "medical student" identity (choice C is wrong).

Which of the following concepts is most closely related to the rationale behind the study's method of measuring automatic association? A) Cognitive dissonance B) Illusory correlation C) Social cognitive theory D) Social schemas

D. Social schemas are cognitive structures that guide the information processing of ideas about categories of social events and people. When a social schema is made more accessible through priming, it can be activated and used more quickly in a particular situation. In this study, subjects' social schemas regarding African Americans were made accessible by priming them with stereotypic African American trait words. This is evident by the control group subjects' significantly faster categorization of African American photos after being primed with stereotypic trait words (choice D is correct). According to Festinger's cognitive dissonance theory, inconsistency among attitudes (cognitive dissonance) propels people in the direction of attitude change. Cognitive dissonance does not play a role in affecting the speed at which a subject categorizes photos after being primed with certain trait words (choice A is wrong). Illusory correlation refers to people's overestimation of instances that support their beliefs about the association between two things. The study does not involve having subjects estimate the number of times they have seen a person act in a way that is consistent with the stereotypes for that person (choice B is wrong). Social cognitive theory refers to Albert Bandura's theory that learning occurs in a social context with a dynamic and reciprocal interaction of the person, environment, and behavior. The study did not involve having participants learn behaviors through observing others' behavior (choice C is wrong).

How would the functionalist sociological perspective view chronic illness? The distribution of chronic diseases reflects social patterns in the distribution of wealth and power. Disease is a form of deviant behavior because it interrupts the contributions of people to their society. The medical profession is important to regulate the production capabilities of those with chronic conditions. A) II only B) III only C) I and II only D) II and III only

D. Structural functionalism is centered on the concept of society as a living organism. According to this sociological model, each individual social structure has a distinct purpose that is important for the function of the greater society. Item I is false: Health disparities as a result of social inequalities in the distribution of resources is of interest to conflict theorists, rather than functionalists (choice C can be eliminated). Conflict theory is centered on the concept of social competition for limited resources, such as access to health care. Item II is true: Deviance is defined as the violation of social standards for conduct. Because structural functionalism is concerned with the contributions to the greater social structure, functionalists consider disease to cause deviant behavior because it inhibits individual contributions, thus having a negative impact on social order (choice B can be eliminated). Item III is true: The importance of social maintenance in structural functionalism creates the need for sources of regulation, such as medical professionals with authoritative roles in health care (choice A can be eliminated and choice D is the correct answer). Functionalists would argue the important of medical professionals for two reasons. First, medical professionals can assist in the management of chronic illness, thus allowing the individual to continue with their productive contributions to society. Second, in the case of debilitating conditions, the structure of health care can create a separation between the acceptable and unacceptable. The behaviors of those unable to contribute to sustaining social order are regulated, and those with chronic illness are expected to conform to the exceptions of medical professionals in order to maintain structure.

When asked to rate what contributed to their desirability ratings, many daters rated factors such as quality of conversation, eye contact, and mutual interests as being important. Few, however, mentioned staying seated or rotating (doing the approaching) as being important. This is an example of: A) a self-fulfilling prophecy. B) stereotypical prejudice. C) socialized identity formation. D) the fundamental attribution error.

D. The fundamental attribution error is the tendency of people to overemphasize personal attributes or an individual's behavior in a situation rather than focusing on external situational factors, as in the question stem (choice D is correct). Self-fulfilling prophecy is when an expectation of an event leads directly or indirectly to that event happening (choice A is wrong). Stereotypical prejudice is a baseless, negative view of members of a social group based on non-factual preconceptions of the characteristics of the group (choice B is wrong). Socialized identity formation is the establishment of a distinct personality based on interactions with others (choice C is wrong).

For children who experience an avoidant attachment style growing up, parents do not make themselves available emotionally and the child responds by withdrawing. Research shows that children who demonstrate an avoidant style did not connect with a parent emotionally. According to the third paragraph, what could account for this deficit? A) Low level of satisfaction and an "out of sync" relationship B) Inadequate social superstructure C) A poor support network within the family D) Lack of physiological co-regulation in the mother-child dyad

D. The parents in the question stem fail to connect with the child emotionally, leading to the avoidant attachment style. In the passage, physiological co-regulation of emotion is the mechanism through which the mother and child connect emotionally (choice D is correct). Relationship satisfaction and being "in sync" is discussed in the passage regarding romantic relationship, not parent-child interactions (choice A is wrong). In Marxist theory, the superstructure is the collection of social organizations and groups that form the society's structures of power (choice B is wrong). A support network, though important for a child's upbringing, does not specifically deal with emotional interaction between a parent and child as described in the question stem (choice C is wrong).

Suppose a fifty-year-old man takes a polygraph as part of an insurance investigation. No deception is detected in response to any question, even when his answers are obviously and objectively false (e.g., he asserts that he was present at an event that took place before he was born). Assuming that the polygraph is in fact highly accurate at measuring the physiological indicators of stress associated with lying, which one of the following conditions, if established, would help to explain the results of this man's polygraph? I. Schizophrenia II. Antisocial personality disorder III. Alzheimer's disease A) I only B) I and II only C) II and III only D) I, II, and III

D. The polygraph purports to measure the physiological effects of stress (presumably due to guilt) that are associated with lying. Accordingly, the individual being tested must actually be intending to deceive, not simply stating an untruth. Item I is true: schizophrenics often suffer from delusions that they firmly believe no matter how bizarre or implausible; therefore, if this man is suffering from schizophrenia it is highly plausible that his untruths would not cause any physiological stress that could be detected by a polygraph (choice C can be eliminated). Item II is true: those with antisocial personality disorder clearly know full well that they are lying. However, since they experience little or no guilt about the lies, they may not show physiological signs of stress when they lie (choice A can be eliminated). Item III is true: individuals afflicted with brain damage or a dementia, such as Alzheimer's disease, often engage in confabulation in order to fill in the gaps left by lost memories. As with delusions, these false memories are honestly believed, no matter how strange or unlikely (choice B can be eliminated; choice D is correct).

Recently, a married couple experienced mortgage foreclosure, which has resulted in negative thoughts and behaviors from the husband who feels responsible, thus creating significant strain in the relationship. The pair decided to pursue professional counseling to address this strain. The therapist approached sessions with the intention of helping the husband relearn these undesired thoughts and behaviors through reconditioning and other general methods of practice. This technique is described as: A) psychoanalytic approach. B) exposure and response prevention approach. C) humanistic approach. D) cognitive behavioral approach.

D. The problematic behavior described includes negative, self-defeating thoughts. This assumed problem is best addressed through cognitive behavioral therapy, or CBT (choice D is correct). Through techniques such as those described in the question stem, CBT can help individuals learn healthier methods of thinking. Psychoanalytical therapy addresses problems that are often unconscious and/or stem from childhood experiences with the goal of reducing anxiety through self-insight; this is accomplished by the therapist analyzing the patient's past (choice A is wrong). Exposure and response prevention, a specific method of CBT, addresses problematic behaviors, such as obsessive-compulsive behaviors, with the goal of helping individuals discontinue these behaviors; this is accomplished by exposing clients to their fears and preventing stressful responses (choice B is wrong). Humanistic therapy addresses problems that prevent self-understanding with the goal of personal growth through self-insight; this is accomplished by establishing a relationship between the therapist and the client that is based on trust (choice C is wrong).

Gentrification is best described as an example of which form of demographic change? A) Urban blight B) Urban decline C) Urban growth D) Urban renewal

D. The process through which urban communities are redefined, called gentrification, is most often the result of urban renewal efforts (choice D is correct). This calls for the redevelopment of urban areas, and gentrification, in particular, suggests a shift in the communities through economic developments that attract wealthier businesses and therefore residents. This is similar to the concept of urban growth, but gentrification is specific to the revitalization of existing urban centers; urban growth is a more ambiguous concept and most often studies of urban growth are concerned with the initial growth of cities as a result of historic industrialization (choice C is wrong). However, the passage states that gentrification is the opposite of urban decline as it concerns the process through which people re-enter cities (choice B is wrong). Urban blight is one of the results of urban decline in which cities that were once functioning rot to a state of disrepair; this is exemplified through "ruin porn" that features desolate landscapes (choice A is wrong).

Socialized medicine is a term used to describe government regulation of health care. This public administration of health care is funded through taxation. This is also often referred to as universal health care. In the United States, there are some socialized insurance programs, such as military medicine. However, despite the implementation of the Affordable Care Act, private companies continue to provide most of the nation's health care. Public opinion has been slow to accept the notion of universal health care, noted through opposition to presidential reform efforts during the Truman, Clinton, and Obama administrations. This hesitation could be attributed to several factors, such as agreement with conservative critics, which best matches the approach to inequalities described in the theories of: A) Emile Durkheim. B) Karl Marx. C) Ludwig Gumplowicz. D) Max Weber.

D. The theoretical perspective most concerned with social inequalities is the conflict theory. Classical sociologists associated with this theory include Karl Marx, Ludwig Gumplowicz, and Max Weber. However, Emile Durkheim is more associated with structural functionalism, which is focused on contributions to social stability (choice A is wrong). The persistent opposition to forms of socialized medicine best reflect the theories of Max Weber (choice D is correct). As opposed to the other theorists, Weber argued that the presence of inequalities would not necessitate the collapse of capitalism. He suggested that responses to inequalities are moderated through additional social factors, such as agreement with authority figures (e.g., public political figures). The persistent opposition, however, challenges the theories of Karl Marx (choice B is wrong). Marx argued that social inequalities, and subsequent conflict and internal tensions as a result of power differentials, would lead to the rise of socialism. The Marxist perspective then suggests the rise of socialized medicine, as opposed to the continuation of capitalistic private systems. Finally, the theories of Ludwig Gumplowicz focused on cultural and ethnic conflicts that are not relevant to the question (choice C is wrong).

The theoretical paradigm LEAST suited for micro-sociological studies is the: A) constructionist perspective. B) symbolic interactionist perspective. C) dramaturgical perspective. D) conflict theory perspective.

D. Theories with a micro-level orientation are concerned with the nature of social interactions on the scale of the individual; theories with macro-level orientation, in contrast, are concerned with the broader social structure. The main micro-sociological perspective is symbolic interactionism, which includes the dramaturgical approach (choices B and C are suited for micro-sociological studies and can be eliminated). Social constructionism has both macro and micro versions, which reflects the fact that social realities are constructed through both greater social structures and face-to-face social interactions (choice A is suited for micro-sociological studies and can be eliminated). Conflict theory, however, is a macro-sociological perspective, concerned with social inequalities as a result of the greater social structure, which is a particular concern in capitalist societies (choice D is not suited for micro-sociological studies and is the correct answer).

It is common for people with mental disorders to suffer the negative consequences of social stigmas. This has serious effects on the individual and their families as stigma discourages people from finding help and thus makes it difficult to secure the appropriate treatment. Which of the following is an example of a social stigma associated with mental illness? A) Those with depression don't care enough about their health since happiness is a choice. B) Those with anorexia nervosa are too thin and it is not at all attractive. C) Those with bipolar disorder cannot be competent doctors and should not be hired. D) Those with schizophrenia are "crazy" or "psycho."

D. Those with mental health disorders are often referred to as "insane" or other negative terms; these demeaning labels are known as social stigma (choice D is correct). Modern societies continue to assign these labels to entire groups based on behavioral qualities that are considered deviant, as exemplified here, and these labels have serious implications for their targets. Stereotypes are oversimplified ideas, positive or negative, about entire groups of people based on some shared characteristic. For example, a common stereotype of those with mental illness is that it is a choice for those with depression to be unsatisfied (choice A is wrong). Prejudice involves thoughts, positive or negative, about entire groups based on some shared characteristic. For example, a common prejudice against those with mental illness is that those with anorexia nervosa are too thin in comparison to the social ideal (choice B is wrong). Discrimination involves actions against entire groups based on some shared characteristic. For example, a common form of discrimination against those with mental illness occurs in the hiring process for jobs (choice C is wrong).


Ensembles d'études connexes

Living Environment State Lab Questions

View Set

metric prefixes (smallest to largest)

View Set

Policy Provisions and Contract Law

View Set

French Phonetic Transcription Practice

View Set

Developing Effective Teams: Practice Milestone

View Set

Chapter One: Completing the application, underwriting and delivering the policy (QUIZ)

View Set